You are on page 1of 175
ELECTRICAL ENGINEERING PROBLEMS & SOLUTIONS Ninth Edition Lincoln D, Jones, MS, PE Elec. Eng. Ml Ideal review for the breadth/depth exam | MeN ee roreN lee) (ata eer) tela Be cee ake a tag 22 SER Saye W My mime [Pe HES Rar I Oy | “This publication is designed to peovide accurate and authoritative information in regard the subject matter covered. It is told withthe understanding thatthe publicher is not engaged in rendering legal, accounting, oF other professional service. If legal advice ar other expert assistance is required, the services of a competent professional person should be sought President: Roy Lipner Vice-President of Product Development and Publishing: Evan M. Butterfield Editorial Project Manager: Laurie McGuire Director of Production: Danicl Frey Creative Director: Lucy Fenkins ‘Copyright 2005 by Kaplan® AEC Education, a division of Dearborn Financial Publishing. tac.” Published by Keplan® AEC Education, adivision of Dearborn Financial Publishing, I 1 Kaplan Professional Company® 30 South Wacker Drive ‘Chicage, IL 60606-7481 (12) 836-4400 ww aikaplanaecengivieeritg.com All rights reserved. The vext of this publication, or any part thereof, may no be reproduced in any manner whatsoever without writtes permission in writing from the publisher, Print the United States of America, a 0607 1 9 8 7 6 5 4 3 27 CONTENTS |— Introduction v Fundamental Concepts and Techniques 1 PROBLEMS SOLLTONS 12 BECOMMENDED REFERENCES 41 CHAPTER 5 Control Systems 113 LSHAPTEA 5 SOLUTIONS 119 Cuaptea « | Computing 135 PANeLeMs 135 SOLUTIONS 129 Digital m_ 145 PROBLEMS 145 SOLUMONS 150 CHAPTER 8 Communication Systems 159 BROS EMS 15S ‘SOLUTIONS 165 BECOMMENDED REFERENCES 174 CHAPTER @ Biomedical Instrumentation and Safety 175 PROBLEMS 75 SOLUTIONS 177 APPENDIX A Engineering Economics 179 Urheberrechilich geschiitztes Material Introduction Donald G. Newnan OUTLINE HOW TO USE THIS BOOK Vv BECOMING A PROFESSIONAL ENGINEER VI Education @ Fundamentals of Engineering (FE/EIT) Exam » Experience = Professional Engineer Exam ELECTRICAL ENGINEERING PROFESSIONAL ENGINEER EXAM Vi Examination Gevelopment = Examination Structure m Exam Dates m Exam Procedure = Preparing for and Taking the Exam » Exam Day Preparations m What to Take to the Exam HOW TO USE THIS BOOK Electrical Engineering: Problems & Solutions ond its companion texts form a three- Sep approach lo preparing for the Principles and Practice of Engineering (PE) exam: m Electrical Engineering: License Review contains the conceptual review of electrical engineering topics for the exam, including key terms, equations, analytical methods and reference data, Because it does not contain problems and solutions, the book can be brought into the opea-book PE exami as one of your references. © Bectrical Engineering: Problems & Solutions pr 3 problems for you to solve in order to test your understanding of concepts and techniques, Ideally, you should solve these problems after completing your conceptual review. ‘Then, compare your solution to the detailed solutions provided, to-get a sense ‘of how well you have mastered the content and what topics you may want to review further, = Fleciricat & Computer Engineering: Sample Exam provides complete morn- g and afternoon exam sections so that you can simulate the experience of taking the PE test within its actual time constraints and with questions (hat match the test format, Take the sample exam after you're satisfied with your review of concepts and problem-solving techniques, to test your readiness for the real exam. wt Intreduction BECOMING A PROFESSIONAL ENGINEER To achieve registration as a professional engineer there are four distinet steps: (1) education, (2) the Fundamentals of Engineering/Engineer-In-Training (FE/EIT) exam, (3) professional experience, and (4) the professional engineer (PE) exam, more formally known as the Principles and Practice of Engineering Exam. These steps are described in the following sections. Education The obvious appropriate education is a B.S. degree in electrical engineering from an accredited college or university. This is not an absolute requirement. Alternative, but less acceptable, education is a B.S. degree in something other than electrical engincering, or a degree from a non-accredited institution, or faur years of cdu- cation but no degree. Fundamentals of Engineering (FE/EIT) Exam Most people are required to: take and pass this eight-hour multiple-choice exam- ination, Different states call it by different names (Fundamentals of Engineering, E.LT,, or Intern Engineer), but the exam is the same in all states. It is prepared and graded by the National Council of Examiners for Engineering and Surveying (NCEES), Review materials for this exam are found in other Engineering Press books such as Fundamentals of Engineering: FE Exam Preparation. Experience ‘Typically one must have four years of acceptable experience before being permit- ted to take the Professional Engineer exam (California requires only two years). Both the length and character of the experience will be examined, It may, of course, take mare than four years to acquire four years of acceptable experience, Professional Engineer Exam The second national exam is called Principles and Practice of Engineering by NCEES, but just about everyone else calls it the Professional Engineer or PE. exam. All states, plus Guam, the District of Columbia, and Puerto Rico, use the same NCEES exam. ELECTRICAL ENGINEERING PROFESSIONAL ENGINEER EXAM The reason for passing laws regulating the practice of civil engineering is to protect the public from incompetent practitioners. Most states require engineers working ‘0n projects involving public safety to be registered, or (o work under the supervision of a registered professional engineer, In.addition, many private companies encourage ‘of require engineers in their employ to pursue registration as a matter of professional development. Engineers in private practice, who wish to consult or serve as expert witnesses, typically also must be registered. There is no national registration law; is based on individual state laws and is administered by boards of registration in each of the states. A listing of the state boards is in Table 1.1. Biectrical Engneenng Professional Engneer Exam Table 1.1 State boards of registration for engineers State/Territory Web site ‘Telephone worwidecd state.ak usloce/paek nm (07) 485-1676 wivwibels.stateal.us (334) 242-8568 wewwistateanusipels (801) 682-2824 worwastate.arusipels (602) 364-4930 dea.ca.gov/pelsirontacts/aim (916) 263-2240 dova-sute.co.usiengineers surveyors (303) 894-7788 stalectusidep (806) 713-6145 (202) 442-4320 wwwadape org (3012) 368-6708 wow ltpe.or (889) $21-0500 ‘www ans tate ga astplbipels! (478) 207-1450 www.gram-peals ong (671) 626-3138 werw.hawaii,gowidsca/pb (808) $86-2702 \warw.ia us/govermmentioom (515) 281-4126 \wevw.state.id.us/ipelsfindes.tnm (208) 334-3860 worw kprstate hs (207) 785-0877 ‘ww w.in pow/plavbande/engineers (317) 232-2980 KS wwrmaccesskansas orprksbtp (785) 296-3053 KY wwwkyboctsstate hy.s (502) 73-2680 LA wor lapels.com (225) 925-6291 MA worw.statemaureg (617) 727-9957 MD worwadllestate:md.us (410) 230-63. ME \werw.professionsts maineuss.com (207) 287-3216 MI ‘worw.michi gan gov/eisiD,1607.7-154- (S17) 281-9253 10557_12992_14016—00hum.us MN ‘worwactslagid state.mn.us (651) 206-2388 Mo wwrw:prmo.gov/apelsla asp (S73) 751-0087 MP (011) 4670) 234-5897 Ms ‘worw:pepls.state ms.us (601) 389-6160 Mt ‘wwuidiscoveringmontanacomdli/babllicense! (406) 881-2367 bsd_boantipel_board/board_page. htm NC worwinebels.ong (919-881-4000 ND warwindpelsboard org (701) 258-0786 NE wore. state ne. us (902-471-202 NH worw.state.nhustbeard/nonte htm (603) 271-2219 NI ‘wsrw.stte.s (973) 504-6460 NM ‘wervstate.nm asfpepsboant {S18} 827-7561 NV worwboe, state 1.08 (775) 688-1231 NY wawopanysed.gov (518) 474-3846 a8i7 x 140 OH worwcohiopeps. on (614) 4603651 OK worwpelsatate ok! (405) 521-2874 OR wwrwosbeels org (503) 362-2666 mA worw.don state pa. usieng (717) 783-7089 PR P.O. Box 3271, San Juan 00808 (787) 722-2122 n wow bap state Fs (401) 222-2565 sc warwilin.state.sc.uw/POLAEngineers (803) 896.4422 (Continued) val Table 1.1 State boards of registration tor engineers (Continued) State/Tereitary Web site ‘Telephone sp ‘worw.state sd.usdolboandsfengineer (605) 394-2510 1s ‘www. state. tn usicommerce/asrdvse (618) 741-3221 ™ ‘wwrvethpe.state.te.us (512) 440-723 ur ‘www.dopLutah go (801) 530-6632 YA ‘worw.state-va.uvdope (804) 367-8514 vl ‘wwwialledgov.vi/peo-aels.tm (340) 773-2225 \T ‘www vtprofessionals.org (802) 828-3256 Wa ‘wwwedol.wa goviengineersengfront. him (360) 6644-1595 wi ‘worwedel state wi.us (608) 261-7096 wy ‘wow ype one (304) 558-3559 wy ‘wornantds.uwyo.cdn/wrdvborpefborpe.btral (307) 77-6155 Examination Development Initially the states wrote their own examinations, but beginning in 1966 the NCEES ‘took over the task for some of the states, Now the NCEES exams are used by all states, This greatly eases the ability of an engineer to move from one state 10 another and achieve registration in the new state, The development of the engineering exams is the responsibility of the NCEES ‘Committee on Examinations for Professional Engineers. The committee is composed ‘of people from industry, consulting. and education, plus consultants and subject ‘matter experts, The starting point for the exam is a task analysis survey, which NCEES does at roughly 5- to 10-year intervals. People in industry, consulting, and ‘education are surveyed to determine what civil engineers do and what knowledge is needed. From this NCEES develops what it calls a “matrix of knowledge” that forms: ‘the basis for the exam structure described in the neat section. ‘The actual exam questions are prepared by the NCEES committee members, subject matter experts, and other volunicers, All people participating must hold professional registration. Using workshop meetings and correspondence by m: the questions are written and circulated for review. The problems relate to current professional situations. They are structured to quickly orient one to the require- ‘ments, so that the examinee can judge whether he or she can successfully solve it. Although based on an understanding of engineering fundamentals, the problems sequire the application of practical professional judgment and insight, Examination Structure ‘The morning breadth exam consists of 40 multiple-choice questions covering the following areas of electrical engineering (relative exam weight for each topic is shown in parentheses): Basic electrical engineering (45%). This area encompasses economies, ethics, professional practice, safety, electric circuits, electric and magnetic theory and applications, and digital logic. m Electronics, electronic circuits and components (20%). @ Controls and communication systems (15%) mm Power (20%) You will have four hours to complete the breadth exam. Electrical Engineenng Frofessona Engneer Exam, x ‘The aftemoon depth exam is actually three exams; you choose the depth exam, you wish to take, The depth exams are: = Computers & Electronics, Controls, and Communications = Power Clearly, you should choose the exam that best matches your training and profes- sional practice. You will have four hours ta answer the 40 multiple-choice ques- tions that make up the depth exam. Both the breadth and depth questions include four possible answers (A, B, C.D) and are objectively scored by computer. For more information on the topies and subtopics and their relative weights ‘on the breadth and depth portions, visit the NCEES Web site at www.ncees.org. Exam Dates ‘The National Council of Examiners for Engineering and Surveying (NCEES) pre- pares Professional Engineer exams for use on a Friday in April and October of each year, Some: state boards administer the exam twice a year in their state, whereas others offer the exam once a year. The scheduled exam dates are: April October 200s 18 B 006 2 u 207-20 %6 208 on 4 People seeking to take @ particular exam must apply to the state board several months in advance. Exam Procedure Before the moming four-hour session begins, the proctors pass out an exam booklet and solutions pamphlet to each examinee, ‘The solution pamphlet contains grid sheets on right-hand pages. Only the work on these grid sheets will be graded, The left-hand pages are blank and are to be used forseratch paper. The scratchwork will not be considered in the scoring. If you finish more than 30 minutes early, you may turn in the booklets and leave. In the last 30 minutes, however, you must remain to the end to ensure a quiet environment for all those still working and the orderly collection of materials. The aftemoon session will begin following a one-hour lunch break. The afternaon exam booklet will be distributed along with an answer sheet. Preparing for and Taking the Exam Give yourself time to prepare for the exam in a calm and unburied way, Many candidates like to begin several months before the actual exam. Target a number ‘of hours per day or week that you will study, and reserve blocks of time for doing so. Creating a review schedule on a topic-by-topic basis is a good idea, Remember to allow time for both reviewing concepts and solving practice problems. x Introduction In addition to review work that you do on your own, you may want to join a study group or take a review course, A group study environment might help you stay committed (o a study plan and schedule. Group members can create additional Practice problems for one another and share tips and tricks. ‘You may Want o prioritize the time you spend reviewing specific topics according to their telative weight on the exam, as identified by NCEES, or by your ateas of relative strength and weakness, People familiar with the psychology of exam taking have several suggestions for people as they prepare to take an exam, 1. Exam taking involves really, two skills, One is the skill of illustrating know!- edge that you know. The other is the skill of exam taking. The first may be enhanced by a systematic review of the technical material, Exam-taking skills, on the other hand, may be improved by practice with similar problems pre- sented in the exam format, 2, Since there is no deduction for guessing on the multiple choice problems, answers should be given for all of them. Even when one is going to guess, a logical approach is to aitempt to first climinate one or two of the four alter- natives. If this can be done, the chance of selecting acorrect answer obviously improves from I in 4 to 1 in 3 or 1 in 2. 3. Plan ahead with a strategy, Which is your strongest area? Can you expect to see several problems in this area? What about your second strongest aren? What will you do if you still must find problems in other areas? 4, Plan ahead with a time allocation, Compute how much time you will allow for each of the subject areas in the breadth exam and the relevant topics in the depth exam, You might allocate a little less time per prablem for those arcas in which you are most proficient, leaving a little more time in subjects that are dificult for you. Your time plan should include a reserve block for especially difficull problems, for checking your scoring sheet, and {o make last-minute guesses on problems you did not work. Your strategy might also include time allotments for two passes through the exam—the first to work all problems for which answers are obvious to you, and the second to return to the more complex, time-consuming problems and the ones at which you might need to guess, A time plan gives you the confidence of being in control and keeps you from making the serious mistake of misallocation of time in the exam, 5, Read all four multiple-choice answers before making a selection, An answer in a multiple-choice question is sometimes a plausible decoy—not the best answer. 6, Do not change an answer unless you are absolutely certain you have made a mistake, Your first reaction is likely to be correct. 7, Do not sit next to a friend, a window, or other potential distractions. Exam Day Preparations The exam day will be a stressful and tring one. This will be no day to have unpleasant surprises. For this reason we suggest that an advance visit be made to the examination site. Try to determine such items as 1, How much time should I allow for travel to the exam on that day? Plan to arrive about 15 minutes early. That way you will have ample time, but not Blectnical Engineering Professional Engineer Exam xl too much time. Arriving too early, and mingling with others who also are anxious, will increase your anxiety and nervousness. . Where will I park’? 3. How does the exam site look? Thave ample workspace? Where will 1 stack my reference materials? Will it be overly bright (sunglasses), cold (sweater), or noisy (earplugs)? Would a cushion make the chair more comfortable? 4. Where are the drinking fountain, lavatory facilities, pay phone? 5. What about foad? Should I take something along for energy in the exam? A ‘bag lunch during the break probably makes sense What to Take to the Exam ‘The NCEES guidelines say you may bring only the following reference materials and aids into the cxamination room for your personal usc: 1, Handbooks and textbooks, including the applicable design standards. 2, Bound reference materials, provided the materials remain bound during the entire examination. The NCEES defines “bound” as books or materials fas- tened securely in their covers by fasteners that penetrate all papers. Examples are ring binders, spiral binders and notebooks, plastic snap binders, brads, screw posts, and so on. 3, Battery-operated, silent, nonprinting, noncommunicating calculators. Begin- ning with the April 2004 exam, NCEES has implemented a more stringent policy regarding permitted calculators. For more details, see the NCEES website (www.ncees.org), which includes the policy and a list of permitted calculators, ‘You also need to determine whether or not your state permits preprogrammed calculators. Bring extra batteries for your calculator just in case; many people ‘feel that bringing a second calculator is also a very good idea. Atone time NCEES had a rule barring “1 toward sample questions and their solutions” in the exam room, This set the stage for restricting some kinds of publications from the exam, Stare beards may adopt the NCEES guidelines, ar adept either more or less restrictive rules. Thus an important step in preparing for the exam is to know what will—and will not—be permitted, We suggest that if possible you obtain a written copy of your state's policy for the specific exam you will be taking, Occasionally there has been con- fusion at individual examination sites, so a copy of the exact applicable policy ‘will not only allow you to carefully and correctly prepare your materials, but will also ensure that the exam proctors will allow all proper materials that you bring to the exam. Asa general rule we recommend that you plan well in advance what books ‘and materials you want to take to the exam. Then they should be obtained promptly so you use the same materials in your review that you will have in the exam. ‘view publications directed principally License Review Books The review books you use to prepare for the exam are good choices to bring to the exam itself. After weeks or months of studying, you will be very familiar with their organization and content, so you'll be able to quickly locate the material you Introduction ‘want to reference during the exam, Keep in mind the caveat just discussc—some state boards will not permit you to bring in review books that consist largely of sample questions and answers. Textbooks If you still have your university textbooks, they are the ones you should use in the exam, unless they are too out of date. To a great extent the books will be like old friends with familiar notation. Bound Reference Materials The NCEES guidelines suggest that you can take any reference materials you wish, so long as you prepare them properly. You could, for example, prepare several volumes of bound reference materials, with each volume intended to cover a particular category of problem. Maybe the most efficient way to use this book would be to cut it up and insert portions of it in your individually prepared bound materials, Use tabs so that specific material can be located quickly. If you do a careful and systematic review of civil engineering, and prepare a lot of well- organized materials, you just may find that you are so well prepared that you will not have left anything of value at home. Other Items In addition to the reference materials just mentioned, you should consider bringing the following to the exam: ® = Ctock—You must have a time plan and a clock or wristwatch. & Exam assignment paperwerk—Take along the letter assigning you to the exam, at the specified location, To prove you are the correet person, also bring something with your name and picture. & Hems suggested by advance visit—If you visit the exam site, you probably discover an item or two that you need 10 add 10 your fis 1 Clothes—Plan to wear comfortable clothes. You probably will do better if you are stightly cool. Box for everything—You need to be able to carry all your materials to the exam and have them conveniently organized at your side. Probably a cardboard box is the answer. CHAPTER /-- Fundamental Concepts and Techniques OUTLINE PROBLEMS 1 SOLUTIONS 12 RECOMMENDED REFERENCES 41 PROBLEMS 1.1 Determine the node voltages V,, V:, and V, in Exhibit 1.1a by setting up the proper nodal equations in matrix form, maniputating the matrices to form the matrix solution for the voltages. 1.2 The 502 Toad is to receive maximum power from the system in Exhibit 1.2, A small series reactance, Z,, is to be placed in series with the load to accomplish, this, Find the proper element for Z, and calculate the power into the 30-9 load. when this element is placed in the circuit. 2 Chapter? Funcamental Concepts and Techniques =} 6 serena) Ns S55 (Assume Z, = 601+ j80, 22 = 30 - /50,f= GOH2) Exhibit 1.28 1.3 Find the current through the 3 ofm resistor for the circuit in Exhibit 13 with a dependent voltage source. Alsa find the power taken from the source. Check the powers being dissipated in the resistors: Do they equal the power taken from the source? Exhibit 1.3, 1.4 Determine the value of i(1), as shown in Exhibit 1.4a, in the steady state, Exhibit 14a 1.5 Find the phasor current / and the phasor vollage drop across each element in Exhibit 1.5, in polar form, using the source as the reference for angles. 4908 o.03H f= 10,0002 10a Exhibit 1.5, Problems: 3 1.6 The frequency of each source is 1000 Hz with the phase relations shown in Exhibit |,6a. Replace this citcuit by its Norton phasor equivalent. #=1000H2 = 100 OSF 02H -——wan—_ |i Exhibit 1.64 1.7 A 2.0 uf capacitor is charged so that it has 100 volts across its terminals. ‘The terminals are suddenly connected through a negligible resistance to the terminals of a 4.0 uf capacitor having no initial charge. a. What are the final steady state voltages across each capacitor? b. What are the initial stored energies of each capacitor? c. What are the final steady state stored energies of each capacitor? 1.8 A 100 ohm resistor is placed in series with the 4.0 uf capacitor in Problem 1.7 so that the charging current will flow through the resistor, The same 2.0 jf capacitor charged to 100 volts is connected to the combination of the 4.0 j1f capacitor and 100 ohm resistor in series with the 4.0 if capacitor having no initial charge. a. What is the time constant of the circuit? b. What are the final steady state voltages across each capacitor? ¢. What are the final steady state stored energies of each capacitor? d. How do you explain or account for the results of the energies calculated in (c) above in light of the different resistor power losses? 1.9 The circuit shown in Exhibit 19a is in a steady state, a. What current is drawn from the power source in thi condition? b. What is the analytical expression for the current drawn from the power source after closing switch S? ial steady state Exhibit 1.98, 4 Chapter Fundamental Concepts and Techniques 1.10 The switch in Exhibit 1.10 was closed sufficiently long ago such that the current # (through the source) has reached steady state. The switeh is then opened at time 1 = 10) a. Find the current i just before the switch is opened, that is, at b. the current j just after the switch is opened, that is, at the current as a function of time afier the s find i(t = 4,), where t= (0). (—- ={Ift|: Se nO (0"y itch is opened; that 1, =24 a=ta | a =10 Exhibit 1.102, LLL Por the circuit shown in Exhibit 1.103, determine i,(("), i{0°), energy stored in E, and £; at ¢=0, i,(0) and é(2) for ¢ > 0. 1.12. For the circuit shown in Exhibit 1.12a, find and sketch i; (1). Exhibit 1.120. 1.13. Find the voltage, v, as shown in Exhibit 1.132. The circuit is initially unenergized. Exhibit 1.438 1.14 In Exhibit 1.14, Vy is a 60 Hz source. The goal is to maximize the power delivered to the 108 2 resistor. The 75 Q resistance is internal to the source. X, and X, are reactive elements (capacitors or inductors) to be added. Find appropriate values for these elements so that the power to the load is maximum, Ey] O. 108Close 750 Exnipit 1.14. 1.15. The current flowing in the 200 Q load in Exhibit 1.15a is 165° out of phase with the 20 volt source. Find the coupling coefficient & and the current i(t). s00e by , a “ EQ 2008 1001 aon 3 = 1 Exhibit 1154, 1.16 Find the Thevenin equivalent circuit for the circuit in Exhibit 1.16a at terminals a and b, All resistor values are in ohms. Chapter 1 Fundamental Concepts and Techniques 1.17. Given the circuit in Exhibit 1.170, find a. n for maximum power in the 2-ohm re b, the power in the 2 ohm resistor if n = 4, Exhibit 1.178 The following data applies to problems 1.18 through 1.25. The expression far a wave is given as fit} = WO0.0sin aat + 20cosi3 ant + 90%), ‘Then f{th is the sum of these two waves: Exhibit 118 A sketch of the wave for problems 1.18 through 125 1.18. The de component of the wave, 0), is a, 0.0 b. 10 ec LS d. 20 1,19 The half-period average of the wave, /(N), is a 2.0 b, 3.28 © 5.96 d. 6.36 1.20 121 1.22 123 124 125 1.26 ‘The rms of effective value of the wave, fir), is a. 7.07 b 721 & 7.63 a 8.02 ‘The maximum or peak value of the wave, fi) is a 100 b LS ce. 120 a 14.0 If the wave, f(t), represents a current in amperes that is flowing through a resistor having a resistance of 3.0 ohms, the power loss in walls is a 10 b 156 c. 187 a. 221 If the wave, (0), represents a current in amperes that is flowing through an inductor having an inductance of 0.1 henry, the rms or effective voltage in volts across the inductor is, a, 033 b 0.67 c 083 d. 1.00 If the wave, fli), represents a current in amperes having an angular velocity of 6000 radians per second, passing through a capacitor having negligible losses, with no initial charge, and a capacitance of 0.002 farads, the rms of effective voltage in volts across the capacitor is a, 0.59 b, 0.74 ec 1.00 d. 5.90 An alternating current voltmeter consists of a series connection of an ideal half-wave diode and a D*Arsonval meter, The meter is calibrated to read the rms value of an applied voltage. When the waveform sketched in Exhibit 1.25a is applied, the meter reads 80 volts, What is the peak value of the applied waveform? Vonage Tine Exhibit 1.250 A 30 micro-ampere meter movement with a 2k ohm internal resistance is to be used with a shunt arrangement so as to have full-scale ranges of 10mA, 100mA, 500mA, and 10amps (Exhibit 1.26a), The maximum voltage drop across the input terminals for a full-scale deflection should not exceed 250 istor sizes needed and state any assumptions made, Chapter 1 Fundamental Concepts and Techniques Exhibit 1.268 1.27 Anamplifier of voltage gain equal to 70dB is tobe built by cascading transistor stages that have a gain-bandwidth product of 3 < 10° radians/see. Find the maximum bandwidth that can be achieved using these stages, and the total number of stages required. 1,28 Refer to the cireuit in Exhibit §.28a. (a) If the inductance £ is 10:0 micro henries and the frequency of the applied voltage is 9.55 MHz, determine the Value of the reactance of C so that the circuit will be series resonant, (b) What is the impedance looking into the circuit under these conditions? & A= 1200 Exhibit 1.288 1.20. For the circuit in Exhibit 1.29a, find H(s) = V\(s)V, and all its eritical frequencies. Then find an expression for [H(co)| and determine |H( jm. 19 19 Problems: 2 1.30 Determine the transimpedance V,(S\//,(5) for the network in Exhibit 1.30. x or Y% AG wy RY, o- Exhibit 1.30. 1.31 The network shown in Exhibit 1.31a represents an oscilloscope prabe con- nected 10 an oscilloscope. The components C, and R, represent the input circuitry of the oscilloscope, and C, and X, represent the probe. a. Find the transfer function ¥,/V,(s)- b. Find a relationship among the components that makes the natural response equal to zero for all time. ©. Suppose the excitation, V,(f), is a unit step of voltage. Sketeh ¥,(0) if Gq R Gy gy Gi _& C+, R+R, C4, (R+R, C4+C,) RFR, (a) Exhibit 1.318, ‘The following practice problems have relatively short solutions. When- comparing answers, choose the nearest value to your calculated one. If your solution is a graphical one, remember that valtage and current do not have to be plotted to the same scale and, of course, use a reasonably large plot. ‘When solving problems in the actual exam, a number of tips may help you eliminate certain answer choices and improve your odds. For example, always glance at the answer selections to see the form of the answer, Are the answers separaied widely? If so, two orthree digits may be sufficient, Is-only the magnitude wanted? If sa, you can carry along any vector notation only as far as it is needed. Is it easier to solve graphically? 10 Ghapter 1 Fundamental Concepts and Techniques 1.32 1.33 1.34 1.35 1.36 For a parallel plate capacitor separated by an air gap of 1 cm and with an applied de voltage across the plates of 500 volts, determine the force on an electron mass of 18.2 x 10" kg inserted in the space. The mass of an electron is 9.1 x 107! kg. a 3.2% 10N b 1.6% 104N 91x 10'N d. 1.6% 10? N Assume a point charge of 0.3 10° C at an origin. What is the magnitude of the electric field intensity at a point located 2 meters in the x-direction, 3 meters in the y-direction, and 4 meters in the z-direction away from the origin? a. 500 k¥/m b. SkVim c. 93 kim d. 9.3 MV/m An infinite sheet of charge, with a positive charge density, oF, has an electric field of E = 6/2¢,),, for x > b, If a second sheet of charge with a charge density of cis placed at-a (see Exhibit 1.34), what is the electric field for asx 1400 108= JX, For the conjugate match, the real part of the load must be equal to the imaginary part, thus (108) © 107 (108 C* w= 2060 = 377 T5(1 + 1,66°10°C") = 108 440, C= 265710" 1,66" 10” C= 163 ur Then for complete matching, the reactive parts must cancel each other. Far the load we have LX) = |=149.8) aL = 49.8 = 75+ 49.8)Q 8a be py 70-132 henry. An equally valid solution can be found by assuming the load to be inductive and the source capacitive. Soutions 27 1s Exhibit 1.156 M = KEL, =kvixd = 2k (100 + fl00)/, + j200K7, 200K, + (200 + 400), (1+ ih, + kh = 0. Jd, + (1 + 32 aes wei-—3 24+v3 (0.2) (0.313)290° Ja-20313" +02 = 75° = 002022165" A it) = 20.2 cos(100r + 165°) mA 28 — Chapter 1 Fundamental Concepts and Techniques 1.16 Convert the current source in parallel with the 7.52 resistor to a voltage source in series to get the circuit in Exhibit 1.16b. Exhibit 1.160 Wij) ~ Sis, = 10 ~Sijy + 22.5igy = 75 1 10 Exhibit 1.160 1Oi,, Sig) = 10 Sigg + 12.Sigg = 7S 10 | ie ee 10-5 125-250 4" L, 125) Soutons 2 Exhibit 1.16d. 2 The Thevenin equivalent is as shown i 5a a 128v b Exhibit 146d L.17 a. Find the Thevenin equivalent of the circuit to the left of the transformer (Exhibit 1.176), 180, a Exhibit 1.17 ‘To obtain the maximum power from this cireuit the load across il should. be 18.9. The impedance seen looking into the » tun side of the trans- former is 4°(2) = 32. The impendance seen looking into the n tum. side of the transformer is #° (2). 1°(2) = 18 n= For a = 3, Power in 22 = 50 w . If =4, the impedance seen looking into the transformer in 2.2 is the same as power into transformer, Using current division, 6 _6, 6+as 5 32) = 46.08 w, or calculating power in secondary, p=($xa] (2) = 46.08 w. 30 © Chaptor1 Fundamental Concepts and Techniques, 1.18 a. de component = LF peony 1p 1 pe 7 =f WWsinonaton)+ 1 [ Zoos( Sear + 90°) deve =0+0=0 1.19 c. The half=petiod average of the wave, f(t), is dc half-period average = 2 5 [rode Ns = 2 [[sosinertan ! [[2ee120r+90>a09| ato 340 1 1 =e =2/3- U3) = —[20-4/3)= qitlo 23-23] ql 413) =5.95. Note that there is one more negative than positive third harmonic loop per half-cycle. 1.20 b. The mms or effective value of the wave, f(1), is effective valve of fundamental = F, = = effective value of 3rd harmonic = f= 5 2 100, 4 Fronercina = VF + FE = (ee 2 1.21 ¢, Note from the sketch the fundamental and the 3rd harmonic peaks occur in phase, thus fps i merely: ADjea, = 10-+ 2 = 12. 1,22 b. From Problem 1.20, Tog =f +1) = 7.2L amperes. 22. P=P R= (7.21) (3,00) = 156 watts, Sokwons 31 1.23 ¢. The effective voltage across the inductor is and =F i436 = 0.80. v2 1.24 a. For @ = 6000 radians/second, 10 Ww lth “x10 2102 2B 2 ny 36N2 1.25 The voltage as seen by the D'Arsonval meter (assuming a lossless diode rectifier) from a pure sine wave: ¢ = Eau. sinat Exhibit 1.255 ‘Then: Eng tet #) Epa sinontton +f, don Actual voltage read would be Ej. %; but the meter is calibrated to read E, = (for an rms value}. v2 For the wave shape given, Chapter 1 Fundamental Concepts and Techniques Time Then E, ide 'p “Th oT qo But the meter reads (for a pure sine wave) E,,.., =—7E,,,. Then, for the sawtooth wave, as given v2 80x volts ~Fyen $i) (2 E, Jos» =143,7 vols. Xyoe BO volts is not the truc rms voltage of the given wave form. 1.26 R, can be determined from the voltage limit, At 0.250 volts, the current through the meter must be SOpa when using the 10ma setting ‘Then 0.25 R.+2k 50x10%, R,=3kQ. Now the current through the shunt string of R, + Ry + Ry + Ry must be 10ma = SOA when the voltage is 0.250 volis. 10 x 107 ~ 50x 10 = 9.95mA This is only 0.5% less than 10m. ‘The accuracy of our computations will not be seriously affected by ignoring this small amount when caleulating the value of RAR, HR +R. Then R, +R +R +R 2 2250, - 10mA, Souwons BB Now with the switch in the 100ma position, the equivalent circuit looks like Exhibit 1,26b, A, Ay Exhibit 1.26b R Ay Ay 500 ma —= a, a Exnibt 1.266 Assume Ry -Ry + 2k since Ry + Ry + Ry + Ry = 25 and R, + 2k = Sk. ‘This is at worst a 0.5% error if Ry + Ry + Ry = 0. Then R, +R, +R, So R,=25-2.5=2259. Now with the switch in the 500 mA. position. the circuit looks like Exhibit 1.26. Again, the error of neglecting 2, + R,+ R, + 2K is less than 0.5% so R+R = 2% ** $00 x10 and R,=25-0.5=20. =052 Finally, the switch at the position for 10 amps is shown in Exhibit 1.26d. Ay R, R, Ry 1Wamps —= @) Exhibit 1.260 Chapter 1 Fundamental Concepts. and Techniques 1.27 As before, Ry + RR, 42K. and Ry = 0.5 - 0.025 = 0.4752. The absolute value of these resistors could be calculated without these simplifying assumptions by writing simultaneous equations for both parallel branches of the cireuit in each case, Because resistors are seldom available in better than 1% tolerances, or at best 1/2%, the added labor for this additional aceuraey is not warranted. A vollage gain of T0dB, 2log*? = y, =3162 for optimum bandwidth, the individual stage gain should be 1.65 (see Modern Electronic Circuit Design by David Comer, Addison-Wesley, 1976). Then 1.65* = 3162 nlog 1.65 0.228 Because an integral number of stages is required, we select 17. This leads to an individual stage bandwidth of 3x10" 1.65 BW 182 10° Rad/sec, Bandwidth shrinkage due to cascading is W,,. = (Bandwidth of single wo | ~isanior(2'=1) 6 BW =37.110° Radfsce =p 9 Lx 10° Hz, Souters = 35 1.28 a. Solve the circuit by the admittance method, 72007 1 0833x10" _ i, lis -Hs oR, jx, 0.833107 + j¥, D DR y., = 0.833% 107 + jk, where denominator = D after rationalizing the fraction with the conjugate D=0.693x10%+¥2 using (a—6)(a+6)=a°-b’, ‘The new equivalent circuit is shown in Exhibit 1.28b. Exhibit 1.280 At resonance, Xe X, X,, = 2a = (6.28)(9.55 x 10°)(10 x 10) = 599.7 obs. Thus Xe = $99,7=X,, based on the principles of resonance. Xx, 4 WB = 599.7, then Y., =599.7(0.693% 10% +¥2) 599.7) i, +4.156x10 =0. Solving the second-degree equation by using the standard formuta we obtain sll =4(4.156% 107 (599.7) 2x 599.7 _ l+vi-0.9979 1 “24599711998 = 0.833107 38 = Ghapter 1 Fundamental Concepts and Techniques, where the radical was approximated to be zero, Therefore the reactance requested is x, =; = 1,199.4 ohms, b. Z,#R,, a8 in resonance the only effective part of impedance is the real part of the complex expression. 0833x107 __0.833%107 Dp 0.693 x10 + ¥2 _ 0833x107 0.693 x10 +(0.833x 107)" Z,, = 0.6 * 10? = 600 ohms. 1.29 Exhibit 1.206 VG (s+ Dh = -sta( sors), : -Fh +P +54 Dh= pri yy P42 +25¢1-5° ‘Solutions The critical frequencies are: Zeros: § = 0, 5 3 00 Poles: 2s) + 2s+1=0 2tV4~8 4 -05 £j05 jo jo H(o)= ‘ te? + atl 1-20 + j20 o o [H@)F > = F Vl-20?) +407 Vi+dor" IH(ool _,., occurs when H(w) is real. H(0) is real when | — 2a7' = 0, 1.30 Node Equation at X: VAs) Vi (s)— KV,(S) TR %, “1, 1 R(sC,R, +1) "SCCRR, + HC ,R, + CR, + KICR) +1 Ves) = KV,GS) Vie 1.31 a. Use a voltage divider. My yo Erg 38 Chaptor1 Fundamental Concepts and Techniques b, To make the natural response zero, climinate the pole in {2 by causing it to cancel with the zero. ‘ vet 1 R+R, GR RRC +O.) GIG RAR, GR ‘Thos: oh, 5 + ade yt == R+R, Souioss = 39 2.x 10°" C. Thus the 1.32 b, The “mass” of 2 electrons has a charge Q = electric field is 500 Esa =50x10° WA 0.01 m The force is then F = QE =(3.2x10"")x(50x10")=1.6%10" N, 1.33 €. The magnitude of the length of the resultant vector, R, in the.x, ; z plane is, Dexa = 20 ‘The magnitude of the electric field, E, is p= 203x107) “~ AmeR? — 4n(8.85« 10" )29 = 93,000 Vim. 1.34 €. On the b plane (for that plane alone), E” = (~of2e),,.. and for the nega- tively charged plane at the a plane (again for that plane alone, but acting to the right of a), is the same as before. (z] Vim fo * 35, the value of E,., is 12.6 x Therefore, ESE +E 1.35 d. From a sketch of the vectors in Exhibi 10° Vém. “ Tr 10,0 1m E,4, = 12.63 108 Wim Exhibit 1.35 80° © Chapter Funcamontal Concepts and Techniques F is then found to be ___10x10* 4x(8.85% 10° yr? FaQ.E,, =5% 10° ¥12.6% 10" = 63% 107 N The cross-sectional area of the iron core is tr? = x10 m?. ‘The path length is: (= 27K =2n(5x107)=0.1am. ‘The permeability is ft = 1,4, = 42(107) x LO" = 40x 10%, 7 =390 A+ vm, and because H = mmf/length, mmf = H x length = 390 0, lz = 122.5 A=t. ‘Thus, / = mmf/tums = 122.5/ 100 = 1.225 A. 1.37 c, Assume the wo wires are bundled close together (with respect to the 0.2 meter position). Then the net current to the right is: daa = 5 L= 4 A. The flux density is given by, QU) Grxto7 (4) + Ba HD CERIO KS) 410 Tesla, tar Qaxd2) 1.38 d. The flux through the air gap and the core must be the same. Because there is no fringing, the effective cross-sectional gap area is the same as the core, Thus the B's are the same, For Bogen =0.6 Tesla, H= 400 A+ im = (mmifylength), so TAME aay = 400% 0.35 = 140 A +t Because H_, = 2 = “6 _ “er wy (4m x10) = = ° and mmf, = H length = 0.48 x10 ‘The total mmf = mmf + MUTE = 048% 10° = 140 + 480=620 At, 1 =mméle = ©? = 1.03 A. 600 1.39 a. Assume ade magnet Fone in dynes: F aA Maxwell's equation . (30,000)* 72x10 72x10" (1) Force = 12.5 Ibs/sq. in. pull Force in Ibs/sq, in. Recommended References 41 (2) B= 30,000 lines/sq. in., L = 0.5 in air gap. MM, yay = 0.85 total NI Ecsaq OF Coil = 24 inches V = 70 volts on coil = 0.313 B NI perinch for air gap Haiay Nl expuq = 0.313% 30,000 0.5.2 = 9,400 amp- tums for aie g _ 9400 ~ 0.85 = 2-400 wot 0.85 be wee = 11,050 ampere tums for air gap Nf, = 11,050 ampere tums for air gap and magnetic ekt. Because the coil dimensions are not given, assume 400 square inches of radiating surface and allow 0.7 watts per square inch dissipation at oor. ‘Waits = 400 x 0.7 = 280 watts dissipated 280. watts and [= 7S S™ =4 Ampere coil curent ML 1050 «9763 tums on coil 1 4 2763 24inches 26d van #125526 _ 5960 cir mills No. 14 AWG Magnet wire has 4,107 cir. mills and 2.525 ohms/ L000". No, 15 AWG Magnet wire has 3,257 cir mills and 3,184 ohms/1000', Choose No, |4 AWG Magnet wire. RECOMMENDED REFERENCES Bobrow, Elementary Linear Circuits Analysis, 2" edition, Holt, Rinehart and Winston, 1987. Jones, Lincoln and Howard Smolleck. Electrical Engineering: FE Exam Preparation, 3" Edition, Kaplan AEC. Nilsson, Electric Céreuits, Prentice-Hall, any edition. Machines ‘OUTLINE PROBLEMS 43 SOLUTIONS 51 RECOMMENDED REFERENCES 71 PROBLEMS Nore: Although several of the following problems are much longer than might be expected on the examination, it is suggested that the reader follow them through where appropriate 2.1 An older de shunt motor (Exhibit 2.1a) is intended to be used to drive a Joad whose output power requirement varies between 5 and 13 hp but will only tolerate a small speed variation. The name plate rating of the machine is given as 15 hp, 230 volts, $7.1 A., 1400 rpm. The field circuit resistance is 115Q and its armature resistance is 0.1392. No data or test results are available on its no-load characteristics. ‘The machine needs to be analyzed as to.its suitability for the speed require ‘ments (that is, speed regulation) and its efficiency over the various load require~ ments. It has already been decided that the accuracy of the analysis daes not require taking into account any effect that armature reaction might produce. ‘The data needed for making a judgment on using this machine are a. No-load and 5 hp line currents, b. No-load and 5 hp speeds. c. Efficiency at both § and 15 hp. ‘The following data should be used for Problems 2.2 through 2.11, A $0 kVA transformer rated at 2300/230 volts at 60 Hz is to be tested in a laboratory so that its characteristics can be determined. The standard test requires both an open and short circuit test; the results of the tests are shown in Exhibit 2.2 ‘The open circuit test measures core loss with negligible copper loss. The short circuit test measures the copper loss with negligible core loss. For future and present requirements, the parameters of an equivalent circuit need to be found. In addition, several values of efficiency and voltage regulation are to a3 CHAPTER }— Chapter 2. Machines ol) [] > otradis) {a) Equivalent eireuit {@) Depengent souree meget! Pg BR Pam IER (0) Power law alagram for @ de:shunt motor Exhibit 2.4 determined (the assumption is that the coils are designed such that 2.2 Determine the coil resistance of the high windings side. a 3540 b. 0.605 22 ¢. 0.0065 2 d. 5.20 ‘Open Circuit Test (Core Loss) Short Circuit Test (Cu Loss) I E w 1 E w OSA 20V 187 wats 2L7A sv 570 watts 23 25 26 28 2.10 B Problems 43 Determine the copper power loss. a BTW bv, 757W s. STOW dL kW Determine the core loss. a 1B7W b 757 ©. 570W do 1.5 kW Determine the efficiency of the transformer at full load (assume unity power factor). a. 0.985 b. 0.996 2. 0.990 4. 0.636 Determine the efficiency of the transformer at half load (assume unity power factor). a. 0.985 b. 0.996 c. 0.990 d. 0.636 Find the percent voltage regulation of the transformer for unity power factor. a 98.22% bv LATS c. 96.44% d. -0.62% Find the percent vollage regulation of the transformer for a 0.8 lagging power factor. a 98.29% b. 3.56% c 96.48% 4. =2.62% Find the percent voltage regulation of the transformer for a 0.8 leading power Factor, a. 98.20% bd. 3.56% ec. 96.44% d, 2.62 Determine the no-load standby current when the transformer is connected to its rated source on the high side. a 654 b 65A © 0.654 d. 316A Determine the high side of the steady state current when the transformer is. connected to its rated voltage source on the high side and the low side is inadvertently connected 10 a load of 0.0394 1 pure resistance. a 644 b. OSA ©. 316A 4. 6A For a perfectly operating de generator, the input mechanical power source may be considered to have constant speed over a range of torque require~ ments from no-load to full-load. However, the field current may be consid- ered constant for a particular application. The generator is rated at 15 kW, 240 volts, 62.5 amperes, at a rated speed of 1200 rpm, The armature resistance is known to be 0.2.22, 48 Chapter2 Machines 213 For the stated conditions determine a. The no-load terminal voliage (neglect any armature reaction effects). b. The percent voltage regulation and the input torque taken from the mechanical power source (neglect the rotational losses). ¢. The percent voltage regulation and the input torque taken from the mechanical power source assuming the rotational torque loss is 5.0 N-m. ‘Now assume the speed control mechanism is faulty and while the speed at ‘no-load is still 1,200 rpm, the terminal voltage drops because the load resis- tance is unchanged, and the speed drops to 1000 rpm. Determine d, The terminal voltage and the new power output. ¢. The new input torque taken from the mechanical power source (agi neglect the rotational losses). ‘Two single-phase, 120 volt, 60 Hz motors are being considered for a par- ticular application requiring one motor of ¥s hp and the other motor ¥2 hp. ‘These two motors are located att some distance from a power source so that a low line current is important such that the line voltage drop is negligible. ‘The lower hp one will be a standard split-phase induction motor, Because of the low current constraint, the other motors will be a special type that has a slightly leading power factor (a built-in capacitor in series with one of the windings). The data in Es! 3a have been obtained for these two motors. Power Out Motor ‘Motor Power (Horseporre Efficienc Factor “A 1 60% 0.7 lagging. “3 we 10% 095 tagging Exhibit 2.130 ‘To make an engineering judgment on the suitability of using these two. motors, it will be necessary to find the total power needed, the combined line current, and the combined power factor of these two motors operating in paraltel. In addition, it will be necessary to have a complete, carefully labeled phasor diagram indicating the line voltage (reference), and the vidual and total line currents, 2.14 In an emergency, a de motor must be used as a generator 215 Probes 47 ‘The motor is a cumulative-compound motor; its efficiency is 90%, and its Positive and negative terminals are marked. ‘The cumulative-compound characteristic must be maintained when it is used as-a generator; the rotation must be kept in the same direction and the rpm will be the same. The positive terminal must remain the positive im the operation as a generator. The interpoles must aid commutation in both motor and generator mode. The machine is to deliver the same power to the line in the generator aperation as it took from the line in the motor and the losses in the machine are the same in both cases. ‘Should the shunt-winding connections be changed? ‘Should the compound-winding connections be changed? ‘Should the interpole-winding connections be changed? Should the field resistance be changed? ‘Has the load on the mechanical cluteh of the machine (Exhibit 2.14b) increased or decreased when the operation changed from motor 0 generator if it was running at full capacity as a motor? f. How much does the electrical power transferred at the line connection increase or decrease if the mechanical torque is maintained on the shaft? peoreg Generator (oy A IO KYA, 240 ac voltage source sends power through two ideal transform- es that are separated by a short transmission line to a single-phase induction motor load whose impedance is 4 + j3 ohms (Exhibit 2.15). The first transformer is a 1:2 step-up andthe second is a2:1 step-down; the equivalent series impedance of the interconnecting transmission line is 2 + j2 ohms. The operator is considering purchasing an uc capacitor to connect across the motor load to reduce some of the line loss, Analyze the system using per-unit values to determine the various voltages, currents, and power taken from the source both before and after the capacitor is added. Also, calculate the kVAR rating for the capacitor (to. correct the load power factor to unity). Chapter2 Machines Exhibit 2.15 2.16 217 218 219 An emergency 120/208-volt, 3-phase, 4-wire, 60 Hz generator supplies an ‘extemal circuit. The load on the external circuit consists of 9000 watts of incandescent light connected between line and neutral and evenly distributed among the 3 phases, and a 10-HP. 3-phase air conditioner motor of 83% ccfficiency and 0.707 power factor, 4, Show the phasor diagram of the currents and veltages on the load side of the generator. b. Determine the microfarads of the capacitor to be connected to the generator in order to reduce the generator load current to 105% of that which would flow if the power factor = 1. c. Determine the size of the conduit and wire to be used as a feeder and the required fuse size to protect the feeder when it is run between the generator and its distribution board in the next room, if the load is continuous, and the capacitor calculated in (b) is connected. ‘Two Y-connected induction motors are fed by a4160 V, line-to-line, 3-phase 60 Hz motor-control center 20 feet away. Motor #1 drives a 600 HP com- pressor. The efficiency of the motor is 90%, and its power factor is 0.5. Instruments of motor #2 indicate 1730 kW, 277 amps. a, Show the phasor diagram of the loads, KW and kVA, b. Determine the capacity in microfarads of a wye-connected capacitor bank that is required to correct the power factor of the total lead 10 0,966. ¢. Ifasynchronous motoris install place of motor #2 and used instead of the capacitor bank to achieve the same overall power factor (0.966), what must its power factor be? A 12,500 kVA, 6600 volt, 3600 rp.m., 60 Hz., 3-phase, Y-connected alter- nator has magnetization and short-circuit characteristics curves as shown in Exhibit 2.18, Determine the percentage voltage regulation for 2 0.707 lagging power factor. Let the ac armature resistance be 0.5-ohms and make (and state) any reasonable assumption necessary for your solution. A 10 kVA, 480-120 V, single-phase transformer is to be connected as an autotransformer to connect a 480 ¥ source to a 600 V load, Determine the maximum kVA load that the autotransformer can supply without exceeding the winding current ratings. 223 2.24 225 Problems 49 10 25 in hioarnperes. Armature ehort-cleexst currant Fiokd current, amperos Exhibit 2.18 A B-phase 132-13.2 kV, 15 MVA transformer is supplying a 12 MVA 3-phase toad at 13 kV. The primary windings are connected in delta and the secondary in wye, Determine the primary and secondary winding and line currents. A 2 phase, 60 Hz, two pole, 4.16 kV, wye-connected synchronous generator is to be applied as an emergency power source to supply 500 kW at a power factor of 0.90 lagging. The stator resistance is 0.30 ohmiphase. Rotational losses in the machine are estimated to be 15 kW. Determine the required horsepower and torque of the prime mover, A 3-phase synchronous motor draws 250 A at a line-line voltage of 4000 ‘V and 1.0 power factor from a large system. The machine is wye connected and has a synchronous reactance of 10 ohms/phase. Stator resistance is negligible. If the field current is held constant and the mechanical load is gradually increased, determine the maximum horsepower thal can be deliv= ered without pulling out of synchronism. A 3ephase, 100 hp, 460 V, delta-connected induction motor has a locked rotor code letter F. The motor is started with a wye-delta controller. Estimate the Jargest starting current that the motor will draw from the supply system. A 3-phase induction motor is supplied from a 50 Hz.system, [t runs at 950 rpm at full load, Determine the number of poles, the slip at full load, and frequency of the rotor current at full load. ‘Two 3-phase transformers are to be operated in parallel. One transformer is rated 7500 KVA and has an impedance of 6.5%. The second transformer is rated S000 KVA and has an impedance of 5.5%. Both transformers have the same voltage rating and are set at the same voltage tap. Estimate the maximum kVA load that the transformers can supply without exceeding the rating of cither transformer, The transformer impedances can be assumed to be purely inductive. Chapter 2. Machines 2.26 2.29 2.30 2.32 A single-phase, 25 kVA, 2400-240 V transformer has an equivalent series impedance of 3.45 +j5.75 ohms referred to the high voltage side. The transformer is supplying rated kVA at 240'V and 85% lagging power factor. Determine the percent voltage regulation. ‘The transformer of Problem 2.26 has a no-lead loss of 500 W. For the same loading condition given in Problem 2.26, determine the transformer efficiency. ‘4.460 V. 100 hp, wye-connected induction motor has a locked rotor imped- ance of 0.12 + 0.35 chm/phase. If the motor is started with an autotrans- former set at the 65% voltage tap. determine the current drawn from the supply system at start. A wye-connected synchronous generator is supplying 4000 A au a line-line voltage of 12 kV and unity power factor to a large system, The synchronous reactance is 2.0 ohmsyphase and stator resistance is negligible, If the prime mover set point is unchanged and the field current is increased by 15%, determine the new stator current and power factor, A200 V series de motor has an armature resistance of 0.2 ohm and a field resistance of 0.7 ohms. When operating at rated voliage, the motor draws 50.A. The rotational losses at this operating condition are 180 W. Determine the shaft output horsepower and the efficiency. A separately excited de generator is operating at its full load rating of 150 V, 30.4, and 1800-pm. When the load is removed the output voltage is 170 V, Determine the voltage regulation, armature resistance, and the full load develaped torque. A 3-phase, 200-V, four pole, wye-connected, 60 Hz squirrel cage induction motor has a per phase equivalent circuit shown in Exhibit 2.32, The circuit Parameters are R,=O.15 ohm R,=0.10 chm X,= = 050 chm X,=20 ohms Neglect magnetic and mechanical losses. For a slip of 0.035 determine the output torque. 0K) Exhibit 2.92 2.33 Ifthe motor of Problem 2.32 is started at rated voltage, determine the starting, current and starting torque. SOLUTIONS 21 Soktions BT For the conditions stated (disregard armature reaction since data is unavail- able on no-load characteristics) you can make a simplified equivalentcircuit ‘or a dependent source type model, as shown in Exhibit 2.1. The figure also. shows a power fow diagram to help organize your solution process. ‘The following calculations are obvious for full load conditions using the equivalent circuitdiagram. The shunt field loss (that will be constant) will be ‘The power transfer across the air gap (E, J,) will be 30 x 57.1 — (460 + 395) = 12,278 watts. P,— (Pag + Pare ‘The rotational losses (F & W plus iron loss) will be Pea — Py = 12,278 — (15 x 746) = 1088 watts. ‘The speed and machine-fux-constant relationship is V,-LR, aT aRs 8 N 1400 ip = 0,159 Vipm, ‘The following calculations are needed for no-load conditions. Py Pog tPA Pys 230L, = 12 x 0.13-+ 1088 +0 ‘The no-load armature current equation and solution is a_( 230 1088 oa! oa n (= f+ ous from 1,— 345400" dae Ie (2 ‘e) EN 1769) =4x8308 =1764A or 4.7 A + Choose (Altemative method -assumes P., > P.., * fa = a =473A) 52 Chapter2 Machines The no-load speed is 230-4.7x 0.1. = 1443 pm. 013g Sem Ke The following calculations are needed far a five hp output. The armature current equations and solutions will be 230/, = (2500.13 + 1088+5% 746 hs ‘The speed is Nop = The efficiency at 5 hp out is The efficiency at 15 hp out is Ro 15x 746 Eff (at 1 Shy y= fo 0+ P= SX 746+ 460 + 395 + L088 P,P, tall losses = 0,852, Exhibit 2.1b esutts of the preceding caicuiations, assuming rotational Josses are constant Power Out Speed —‘%Speed Regulation Arm Current —SeEfMicieney 15bp 1400 pm 3.2% 351A 85. Shp 1425 pm se 22a 09.6% ‘Onp 143 rpm 9 47a o Following is an alternate method of solution using the dependent source mode] in Exhibit 2.lb at full load (1400 rpm or 146.5 rad/sec). Solutions 53 The speed and machine-flux constant at 15 hp using rad/sec for spee VLR, &, emad’s)=—7 - Ko Ko" for developed torque Tyee = KOI, = ‘Torque needed for a 15 hp output is 7, = 9 Py = hpx 146 = 152 746= 11190: Ty = fe «= 76.38 N-m. ‘Torque needed for rotational losses is Ty ‘yey 7 Ty = 83.81 — 76.38 = 7.43 Nem. ‘Torques required for a 5 hp output is 748 370 yy o o Ras Exhibit 2.10 Total developed torque needed is For the speed at 5 hp, assume a linear relationship between no-load and full-load speed. ysmxt by wos mx hp x @=— 03X54 151 = 149.5 3730 oT, =743+—— = 324 N= oe = 7434 = 32.4 Nom 54 Chapter? Machines Armature current needed for a5 hp output is i, KD Toy = KDI, BA. ‘These results agree (within the approximations used) for both metheds. From the table of results, a judgmental engincering decision may be made as to the suitability of using this particular motor. 2.2 b. The coils are designed so that RR, where J, and R, are the high-voltage coil current and ue resistance. f, and R, are the low-voltage coil current and resistance. where Ma (tums rato). Ny Since W. alae 13K, +13R, = 570watts and PR, =:R, for good transformer design 570=2FR = 20K, 50,000 _, = PS = 21.7 amps (Rated) 50,000 17 amps (Rated) 0.6050. (High Sid 221.77 (High Side = 579. 0.00605 (Low Side) 22177 2.30 ce. Copper loss = 17, +178, = 570watts 2.4 a. Core loss = [87 watts negleeting the no load exciting current copper Joss which would amount to (6.5) (0.00605) = 0.255 watt output 25 a. Full load efficiency = ————— *—________ {output + cu loss + core loss) Assume power factor =1 i 50,000 50,000 Bfficieney = 5 O00. 870-187” 50,187 89 26 c. Half toad efficiency Assume power Factor = 1 x Etficiency = ——25:000 _ 25.000 _ poy 25,000+ 224187 25,3295 (No Load Voltage — Full Load Voltage) (Full Load Voltage) 2.7 b. Voltage Regulation = Convert the equivalent transformer circuit, referring it to the low-voltage coil. Assume the low-voltage coil has constant voltage E, = 230 volts. Por 570 Quy Resenton = Zz = 52-539 tines = 347 Z, Zing 53 Pit = 55 0.05302 a rc q = [Zigie., ~ Rais...) = ¥e0.053" — (0.012! = 0.0516 ohms Assume rated current /, 17 amps (at unity power factor). ‘A = By + (Regen, teen) = 230+ 217(0.0121+4 j0.053) 2304 2.634 ILS = 232.634 jIL5 392.7.22.84° 232.7230 = 2.70 230 230 = 0.01175 or 1.175 pervent Voltage Regulation 56 = Chapter2 Marhines 2.8 b. Find voltage regulation for Pf. = 0.8 lag. Assume rated current /, = 217 amps. Draw the phasor diagram, Eya Exhibit 2.8 Fy (cost jsin8) +,R, a gg THX ES = 230(0.8-+ j0.6)+ 21710.0121 + 70.053) = 184+ f138+2.63+ j115 = 186.634 /149.5 = 238.2238.7° 238.2-230 8.2 la oe = 0,085 56% Voliage regulation = "5" = 5 = 00356. or 3. 2.9 d, Voltage regulation for power factor = 0.8 lead. Assume f, = 217 amperes, rated current. Ela "ee "Fon. Exhibit 2.9 £1 e Vcos@'+ fsin) 4 EsRigan, + itEY, a Te Fah oe = 230(0.8 — j0.6) +2.63+ j11.5 = 184= j138-+2.63+ j15 = 186,63= j126.5. = 224.234.2° 224-230 230 = 0,0262 or -2.62% Voltage regulation = Solutions = ST 2.10 e, V, 3 rpanmae #2 = 2300 0.65 A (neglecting Cu Loss.) 2, 3538 2d Z MIS" 5.290 (veglecting core loss) -coppoiaite = 9 9 W_ 570 2 uy _Ww_ 570 “VA S217 Xgrcaee ~ Z8in76.8° = 5.29% 0.974 = 5.13 iebsie =1210 cosg = 0.228, ¢=cos 40.228) = 76.8° 0.0394 = 3.9402 Resasrsase = OR = NEW Zyyye = 121+ j9.159-4+3.94 = 5.15 j5.15 V2x5.152 445° Vv 2300 v Niesae ™ 5 = 316 AZ ~45° Wie” Zw VEXSISL +45 (neglecting core loss current) 2.12 a, The no-load terminal voltage (since there iso speed change) is F, (may be given as “bq. It remains the same fora generator, and is given as E, = V,41,R, = 240 + 62.5 x 0. 252.5 volts. b. The percent voltage regulation is PNR. = (Vo Vi pea DOW Gs os = (252.5 — 40} 100/240 = 5.2%. The input torque is found by Tax = Ef, = 252.2 x 62.5 = 15,781 watts, thus, P= EJ,fo = 15,781/](1200 x 22/60] = 125.6 Nem ¢, The voltage regulation is the same (5.2%) but the new required torque is increased by the amount of rotational loss torque, T’=T+5.0= 125.6 +5 = 130.6 N and the new power input requirement is P= T * @= 130.6 x(1,200 « 2x/60) = 17.93 kW. d. E, is directly proportional to speed for a constant field, EJ =E,(1,000/1,200) = 210.4 volts 58 = Chapter 2 Machines: The load current then becomes EJ(R, + Ry). where the original load resistance was 240/62.5 = 3.84 Q, 10.4/0.2 + 3.84) = 52.8 A. 104-528 x2 = 200V. 10.56 kW. e. The new required input torque is easily found as T = Paleo = (210.4 x $2,8)/(1000 x 260) = 11,109/104.6 = 106.1 Nem, 2.13 Using the data given in Exhibit 2.13a, Output Motor A = Vicos@ x Eff. Tn this case HP,y = Faction of Load Teas 146 = 0.2SHP x —” = 186.5 watts HP Views _ 0.25736 lint = 025576 23.7 amp VxE.x PP 120x0.7x0.6 Qutput _ 186.5 h As =. Input Motor. Er E,,., = 120 volts Ouiput Motor B = Vicos@x Eff, 746 = O.5HPx HP = 373 waits 1 = WS K7H6 bee 120% 0.95% 0.7 Output _ 373 EM ~ 07 =4.66amp Input Motor B = = 534 watts Exhibit 2.13 Motor Power Factor VA = @ — Sin@ A 07 lag eas 4s? 0718 STL watts B 0.93 lead 3620 182° (03123 S34 watts ‘Total power = Input power A + Input power B 11 +534 = 845 wats Motor A VARS: Motor B VARS Total VARS = 142 Tag cas® = cos(9.6°) = 0.986 lag 214 a Soktions 5@ (cos45.5° jsin 45.5°)+ J (cos 8.2° + jsin18.2°) .1(0.7 ~ j0.71) + 4,660.95 + j0.3123) = 259- j26244434 fl42=7.02- jl.2 Lamp P.=31l P25 Pray = Py + Py = 845 watts No, the shunt winding should not be changed. The problem clearly states, that direction of rotation and polarity (positive terminal) stay the same. ‘Therefore, regardless of whether we have a motor or generator operation, the direction of the current stays unchanged. The only drawback is the subtractive MMF of shunt and serics field winding, giving a differential compounding. This matter is brought up in the next point. Yes, the compound (series)-winding connections should be changed, because the armature current during generator operations is reversed and now opposes (differentially compounds) the shunt field. To still aid (cumulatively compound) the shunt ficld, the compound (scries} winding should be reversed. No, the interpole-winding connections should not be changed. Interpoles or commutating poles are narrow laminated auxiliary poles placed mid- way between the main poles and the plane of commutation, These interpoles are in series with the armature and are wound to oppose and nullify the armature reaction in the commutating plane. This prevents sparking that might cause flashover and also reduces iron losses in the armature teeth, Changing from motor to generator action, the polarity of the commutating pole automatically changes, with the change of the armature reaction MMF, Therefore, commutation in interpole machines is not affected by a change from motor to generator operation or a change in the direction of rotation. ‘Yes, the field (shunt) resistance should be changed. Because the generator voltage has to be Targer than the terminal voltage due to the obmi voltage drop in the armature winding, the flux has to be increased (E, = K@rpm). Increased g means increased field current. Therefore, the shunt field resis- tance should be decreased. 60 = Chapter2 Machines ©. The load on the mechanical clutch of the machine has increased, ete hich qf t—Piee] exhiit 21ab Rated Powetyr. = Losses + Load on clutch (for motor) Rated Poweraapa: + Losses = Load on clutch (for generator) Because losses remain the same in either mode of aperation, the load on the clutch increases when the machine is operated as a generator. f. Using the line connections as a reference point, due to internal losses, the power delivered to the motor shaft is 90% of the reference or input power. Assuming that the same mechanical torque is maintained, above shaft powwer is now the input (prime mover) of the generator operation. There- fore, using the same 90% efficiency of the machine, the generator output referred to the original elecirical power transferred at the line connec- tions has decreased to 0.90 x 0,90=0.81 or 81%, First the per-unit values need to be defined for each portion of the circuit (without the capacitor), Generating Source Motor-load Vouco = 240°V — 1 pu 10 kVA — I pu — OQ0000/240 = 41.67A — 1 pu--> same 240/41.67 = 5.760 2 + 1 pu—--> same 4 + 735.760 = 0.6944 + j0.5208 pu > same > same ion line, Viwet. 2X 240 = 480 V 1 pu VA ve, = IORVA 1 pu Jot. = 10000/480 = 20.83 — I pu Zrat = 480/20.83 = 23.04 2 — | pu Zane = (2 + j2)723.04 = 0.0868 + [0.0868 pu. The total pu impedance as seen at the source is Zine = Blue + Zyoce = 0.0868 + 0.0868 + 0.6944 + 70.5208 =0,7812 + j0,6076 = 0,9897.237,9° pu 1.0.20°/0.9897.237,9° = 1,0102-37,9° pu 1.0102-37.9° pu x 41.67 A/pu = 42.10 A. Parece = Vics = 240 x 42,10c0s37,9° = 7974 watts. Frcace 2.16 a. Solutions 61 The motar load voltage is Feeear@rncece Virsa 010 x 0.8680 = 0.8767 pu = 0.8767 pu x 240 W/pu = 210.4 V. ‘Check. (089 wats Peggy = PR = 4210 x 21,057 x 2 = 886 watts Pring = Card 2¥ Brie ‘Choose capacitor such that Zp, Z.gocox has only a real part, ‘ne Zot was = W016 = .25 2 — 6.25/5.76 = 1.09 pu ‘The new total pu impedance as seen at the source is Za, = 0.0868 + f0.0868 + 1,09 = 1,177 + 0.0868 pu 180.24,24° 180 x 5.793 = 6.838.24.24° O11, 180.24.24° = 0.8475.24.24° pu 0.8475.2-4.24° x 41.67 Al = Vicos® = 240 % 35.41cos 4.24 Phe = line Ring = G441/2) x 2 = 626.9 watts Procure = teeter ™Rresgy = 44-2° * 4 = 7850 watts. is interesting to compare the new load voltage with the original value (2104 V), Vosetoccap = M(Zcaw ll Zag) = 34.41 6.25 = 221.3 V, Not only is motor voltage higher, for a greater moter output, but the line loss pawer loss is less. Also, the capacitor should have a kVAR voltage rating of 240 volts (when operating with the motor unloaded), so instead of a current of 221.3/(-j8.33) = 26.56A, it should be 240/(-J8.33) = 28.8 A, giving a KVAR of 6.9 KVAR. Designating: /,, =Motor current; Ing =Real component off; faq = Quadra- ture component of 7, = Incandescent lights current; f, = Total Toad current; 1, = Corrected total current; J, = Capacitor current, ‘We obtain the phasor diagram shown in Exhibit 2.16. 9,000. = 28.40" amps. 208 62 Chapter? Machines 1 10x 746 3 208 0.830.707 ‘Thus I, = 25.20" Jog = 252-90" 1, = 25425 + j25 = 50+ j25=56226.5° amps 5.3245" amps = 25+ 725 b. Ifthe power factor= |, the T, reduces to the real component of the above current, i.e. 50.£0°, Therefore, we have to reduce J, = 56 226.5° to 105% of 50 amps = 1 = 52.50 amps at an angle to be determined. From vector diagram (Exhibit 2.16) // = (52.50) ends in point B. AC=25 AB=525*- 50.0? =165 BC=25~16.5=8.5 amps E 208 i, ‘Then: Xe = = =14.1 ohms See 3x1, 3x85 I rd 1 n then C = 377 xia) ~ OH Fiphase. Cosa = 3% 18.7 = 56, 10F = (25+ 25)+ jl6.5 = 52.5218" amps ggg = 25+ Hl65= 30234? amps Rating of wire size: 125% (Ta HAP 25% (30+ 25)= 68.8 amps From copper wire table (National Electrical Code (NEC), Article 310, Table 310-16) for 70 amps continuous current we obtain Size No. 4. Soutons 63 From overcurrent protection table at 30 amps (no 125% factor is needed) +25 = 55 amps. Thus we obtain a fuse size of 120 amps. From conduit table for 4 wires and 55 amps we obtain a conduit size of | 14" — 217 M 600 <0.746= 497KW = SOOKW VW 17 a. lotor #5 x0 a4 = cca wan pf=05, thus @=60° 1000 kv SO0KW _ 500 kVA= —ell “ cos 60° 0.5 90 KVAR = 1000 x sin 60° = 1000 x 0.866 = 866 Exhiott 2.170 Motor #2 kVA = 3 x 4160x277 = 1994 1730 =o0sG=—— =0.866; @=30° af roma KVAR = 2000 x sin 30° = 2000 x 0.5 = 1000 2000 4730 KW ~] ‘2000 kVA 1000 KVAF Exhibit 2.17 b. Total load of motors #1 and #2 KW = 500 + 1730 = 2230 KVAR = 866 + 1000 + 1866 KVA = 42230" +18667 = V8.45%10° = 2907 Actual combined pf = cose = 2230 = 0.767 lag; 8 = 40° 2907 Desired combined power factor = 0.966 lag; 0 = 15° kVA new « 2230. 9309 0.966 BC = Required Icading kVAR = 1866 - 2230 tan 15° = 1866 — 2230 x 0.268 = 1866 — 598 = 1268 44160)? xe He 4160, «eva ” 1268x1000 7 '*6oh™ 1 1 C=——_ = +. QR XX, 6.28K 60K 13.6 195pF 64 = Ghapter2 Machines 2230 KW SS A 898 KVAR 6 1866 KVAR 2807 KVA 1268 kVAR. ce Exhibit 2.176 ¢. Assuming that the synchronous motor has the same efficiency as motor #2, which it replaces, KVAR syn. motor = kVAR motor #1 — kWAR desired = AC = AB = 866 — 598 = 268, 800 Ki 1750 Kv A 598 KVAR 8 ‘208 KVAR ¢ Exhibit 2.170 The synchronous motor alone is shown in Exhibit 2.17e. 1750 kVA, 288 KVARL 1730 KW Exhibit 2.172 268 = 0.1 i709 = °° _ 1730 _ 1730 © e039? 0,988 pf =cos9°= 0.988 kVA 1750 218, Rated | = 12500.000 600 4 66003 Field current necessary to give short-circuit line current, from graph: Ide) = 105 A, Sowvons 68. Terminal voltage (per phase) from grap! 6300 3 Terminal rated voltage (per phase) = 3630 volts. 6600 ho 3800 wolts. (Here. one could find the synchronous reactance or impedance by taking these operating values and defining 3630 V x, = 2880V 1090 4 However, because only one condition is asked for, the synchronous voltage drop is the 3630 volts.) 1Xq~ 9890V {Fl (1090)(0.8) = 5A5V ne Not te:seato Exhibit 2.186. Vp + Rude + IXsh 3800 + 545(cas45° — jsin4S°) + 3630¢cos45° + jsin4S°) 752.4 + j2181.6 = 709629 volts 66 Vir = A= Ye (490) = 2096=3800 : 3800 = 86.7% (100) Note that the calculated no-load voltage is 7096y/3 = 12,290 volts. which is well beyond the range of available field current, ‘Thus the saturated limit of no-load voltage would be between 9000 and 10,000 volts. The saturated value of synchronous reactance would be less than the value used here. 2.19 480 V winding rated current = 10,000/480 = 20.8 A 120 V winding rated current = 10,000/120 = 83.3 A 66 = Chapter2 Machines 2.20 2.22 To get the specified voltages connect the transformer as in Exhibit 2.19. Exhibit 2.10 The largest value for , is 83.3 A, The load is § = 600 x 83,3/1000 = 50 kVA, ‘The secondary line current is /, = —5— = 12,000/3 x 13 = 533 A. wavs Because the secondary is wye connected, this is also the secondary winding current. The secondary rated voltage is 13.2/V3 = 7.62 kV. The winding ratio is 132/7.62 = 17.32, The primary winding current is $33/17.32 = 308 A. ‘The primary line current is V3 x 30.8 = 53.3 A. The stator current f=? = —s771 A. iveosa J3x4.16%09 Stator copper loss = 3/°R = 3 x 77.1? x0, 50 W, Prime mover power = 500 + 5.35 + 15 = 520 ki Fora two pole 60 Hz machine synchronous speed is 3600 rpm and a, = 3600 x 277/60 = 377 radisecond. Prime mover torque = 520/377 = 1.38 kNm. ‘The phase to neutral supply voltage V= 4000/13 = 2309.5 V. ‘The excitation voltage is E= V — jX1 = 2309.5 - j10 x 250 = 3403.52-47.3° V. Solutons OT Maximum power is found from the power-angle equation with the angle set equal to 90°, ; 7 23095 _ 3,358,000 W = 2358 kW, Convert to horsepower, Poyx = 2358/0.746 = 3160 hp. 2.23 From NEC Table 430.7(B) the starting k'VA/hp is in the range 5.0 — 5.59. Use 5,59 to give the largest starting current Starting kVA at rated voltage = 5.59 x 100 = 559 kVA. 359,000 Vax4o0 Using a wye-delta starter the starting current = 701.6/3 = 234 A. = T1.6A, Starting current = 2.24 Synchronous speed is "2 where /= $0 Haand pis the number of poles. For p= 6, n, = 1000 rpm. Because an induction motor runs at a speed slightly below synchronous speed at full load, conclude that the motor has 6 poles. Slip = (a, ~ nin, = (1000 — 950) /1000 = 0.05 Rotor current frequency = sf = 0.05 « 50 = 2.5 He .055 x (7500/5000) = /0.0825 p.u. Equivalent circuit is shown in Exhibit 2.25, nz, fy = (0:065/0.0825) x 1, = 0.788 x I, Exhibit 2.25 68 = Chapter 2 Machines If. 7; is at rated load (7500 kVA), f, = 0.788 p.u., which is $= 0.788 x 7500 = 5910 kVA. It is overloaded. IFT, is at rated load /,= 5000/7500 = 0.667 px, 1, = 10.788 = 0,667/0.788 = 0.846 py, 5, = 0.846 x 7500 = 6345 kVA ‘Total load = 5000 + 6345 = 11,345 kVA 2.26 Equivalent circuit is shown in Exhibit 2.26. — ay a a . ve Vs Exhibit 2.28 2400/240 = 10 24020 V 25,000/240 £-31.79° = 104,231.79" A V, + La (R + JX) = 2400 + 10.42 2-31.79" x G.45 + 5.75) V, = 2462.3.20.74° V Secondary full-load voltage = 240 V Secondary no-load voltage = 2462.3/10 = 246.2 ¥ % voltage regulation = 2482-240 , 199 = 2.58% 240 227 Pog = Sc0s0'= 25,000 0,85 = 21,250 W Copper loss = J; R= 10.42" x 3.45 = 375 W Core loss = 500 W P= 21,250 + 375 + 500 = 22,125 W Efficiency = (Pya/P,) x WOO = (21,250/22,125) x 100 = 96% 2.28 See Exhibit 2.28. a an Aq 012+ /0950 Exhibit 2.26 ‘Soltions = 6 Starting current at full voltage = = TFSZA-7LIP A 01347035 Autotransformer ratio = 140.65 = 1,538 With the autotransformer, starting current = 717.8/1.538" = 303.5 A. 2.29 See Exhibit 2.29. Large system V= 12,000! V3 20° = 4928.420° V 1 = 4000.20° A E=V + jXl = 6928.4 + 72 x 4000 = 10,583.1.249.11° V Neglecting the nonlinear effects of saturation, when the field current is creased by 15% the magnitude of the excitation voltage will also crease by 15%, E = 1.15 x 10,583.1 = 12,1706 Because the prime mover does not change, the pawer output does not change, sind" sins = 0.65736 & =41.10° ‘The new stator current is =V _ 12,170,624 1.10° ~ 6928.4 ix sO = A1SS.2-15.66°A. ‘The new power factor is cos 15,66° = 0,963 lagging. 70 — Chapter2 Machines 2.30 See Exhibit Exhibit 2.30 E= 200 = 50x 0.2 40.7) P,, = El = 155 x 50 = 7750 W Pog * Py — rotational. loss = 7750 — 180 = 7570 W Pa. in hp = 75700746 = 10.15 hp 200 x 50 = 10,000 W Efficiency = 7570/10,000 = 0.757 or 75.7% 55V 231 See Ex it 2.31 Exhibit 2.31 Voltage regulation = (170 150/150=0.133 or 13.3% 150 = 170 - 30x Solve for R = 0.67.0 Py, = 170 x 30 = 5100 W © = 1800 x (2n60) = 188.5 rad/sec To, = S1O0188.5 = 27.1 Nm 232 Ryfs = 0.10/0.035 = 2.857 heel =36452-18.4° A 0.15 +2.857 + j(0.50-+ 0.50) BF RyJs = 3 x 36.45? x 2.857 = 11.387 W For a four-pole machine synchronous speed is 1800 rpm. 0,,,. = 188.5 rad/see Trae = Pac guyfOyqe = 11,387/188.5 = 60.4 Nm FecommendedRaterences 71 2.33 At start the slip s = 1. 5S . 0,15 +0,10+ j(0.50 +0.50) Porgy = MR, = 3 121? x 0.10 = 3770 W Tran = Prey! Ogne = 37TO/188.5 = 20 Nm 15.5420 = 5.802-90" A 1+ dy = 121 £-76.0° + 5.80.2-90* = 117.7.2-76.7° A = 1212-76." A RECOMMENDED REFERENCES Chapman, Electric Machinery Fundamentals, 3" edition, McGraw-Hill, 1999. El-Sharkawi, Fundamentals of Electric Drives, Brooks/Cole, 2000, Fitzgerald, Kingsley, and Umans, Electric Machinery, 6° edition, McGraw-Hill, 2002. Hambley, Electrical Engineering Principles and Applications, 3" edition, Prentice Hall, 2005 (Chapters 15, 16, and 17) Kuo, Ausomaric Conieol Systems, Prentice-Hall, any edition. [For more information on the servo motor.} Matsch and Morgan, Electromagnetic and Electromechenicel Machines, 3" edition, Wiley, 1986, Nasar, Electric Machines and Electromechanies, Schaum's Quiline Series, McGraw-Hill, 198). National Electrical Code (NEC), National Fire Protection Association, Inc. Wildi, Electrical Machines, Drives, and Power Systems, 5 edition, Prentice Hall, 2002. ‘Yamayee and Bala, Electromechanical Energy Devices and Power Systems, Wiley, 1994, CHAPTER }— Power Distribution PROBLEMS OUTLINE PROBLEMS 73 SOLUTIONS 77 RECOMMENDED REFERENCES 91 a4 A 3-phase, 60 Hz, 173 KV, 50-mile-long transmission power line delivers complex power to known oad, The line has been tested for its characteristic parameters. These parameters (on a pet-phase basis) are found to be 0.1 obmimile 2.0 millihenry/mile Capacitance = 1.0 x 10 wE/mile For a known load at the end of the line of 75 + j30 (three-phase, total megavolt-amperes}, determine the input (sending) current and the input power necessary to support the complex load. A345 kV power transmission line has two bundled conductors per phase, spaced 18 inches apart horizontally (Exhibit 3.2). The conductor used in the bundle has a self GMD of 0.0403 feet and the phases are spaced horizontally 15 1/2 feet apart. Determine the following a. The self GMD of the bundled conductors. b. ‘The mutual GMD of the line. ¢. The inductive reactance per phase per mile. eee ae ae | eo ce lec fp4—1585—4— 41 se—4—| Exhibit 3.2 74 Chapter 3 Powor Distribution 3.3. Power for a remote building on an industrial is supplied through an existing buried cable from a fixed voltage 60 Hz supply. The load in the remote building consists of lighting and induction motors. During periods of peak demand, when the cable is carrying approxi- mately its rated current, the resi ing steady-state load voltage is well below the desired value because of the characteristics of the load, A small amount of additional constant-speed motor load is anticipated in the near Future, What equipment can be installed at the building to improve the present situation and to permit the additional load? Explain how the equipment you recommend will improve the situation; the use of phasor diagrams is suggested 34 The power in one of its phases. Unity system ay Transtormer bution transmission system in Exhibit 3.4a develops a fault 13MVA, 12KV @H02 Availatie short carcull ae extiot 3.40 1S MVA, 60/12 kV X=6,078 pu Xi=0.200pu Hox Faults °F Determine the fault currents at point “F" for the following conditions: a, Assume the fault is a three-phase one, b. Assume the fault is a single line to ground, 3.5. The voltages of an unbalanced 3-phase supply are V, = (200 + J0)¥, V4 = (-7200)¥ and ¥, =(-100 + j200)V. Connected in star across this supply are three equal impedances of (20+ /10) ohms, There is no connection between the star poi of the A phase current and the three fine currents. and the supply neutral, Evaluate the symmetrical components 3.6. The one-line diagram in Exhibit 3.6a is for a single-phase system involving a voltage source of 240 volt 10 KYA generator that supplies a load through 1:2 step-up transformer, a relatively short transmission line with an imped- ance of | +4 ohms, and a 4:1 step-down transformer. The transformers are assumed to be ideal. 12 a 240¥ TORVA Exhibit 3.62 ean 2 @—3 an —3 : 38 3.9 3.10 3a 312 Probems 75 a. Assume the load is known to be 1 + jl $2. What equivalent load (in ohms) does the generator see? b. Assume the Toad is known to be 1 +l 2, Determine the per unit values of the various system parameters along the system and determine the per unit current through the Toad. €. (Optional for practice) Repeat part 2 except Z, = 2 + j2.2. inally it is planned to furnish a plant load requirement of 1000 hp at 2200 volt, 3-phase, by induction motors operating at 80% power factor and 90% efficiency. a. Find the line current necessary to supply this load and generator capacity. b. Assume that rather than supplying the 1000 hp by induction motors, it is decided to produce 400 hp of this load by a synchronous motor operating in the over-excited leading mode of 85% (assume same effi- ciency as for an induction motor). Find the new total line current requirement and the overall power factor. ¢. If, rather than installing the 400 hp synchronous motor as in Part(b), it is considered feasible 10 use power factor correcting capacitors for the 1000 hp motors in Part (a) to achieve the same power factor correcting ‘as obtained in Part (b). Determine the size (kKVAR) of the capacitors needed. The peak load supplied from a substation 13.8 kW bus is 20 MW at a power factor of 0.85 lagging. Determine the Mvar size of a 3-phase capacitor bank to correct the power factor to 0.95 lagging. A phase 480 V delta-connected capacitor normally draws a line current of 200 A. If'a fuse blows in one phase, determine the current flowing in the other two phases. An induction motor and a synchronous motor are supplied from a common bus, The induction motor takes 100 kVA at 0,75 power factor lagging. The synchronous motor draws 150 kW. The field current of the synchronous motor is adjusted until the combined motor load power factor is unit Determine the power factor of the synchronous motor. A deha-connected 3-phase load of 60 + {30 ohms/phase is supplied by a feeder having an impedance of 0.50 + j0.75 ohmsiphase, Determine the line-tine voltage at the sending of the feeder to provide a Tine-line voltage of 460 ¥ at the load. ‘The available fault current on the 480 V side of the transformer supplying a facility is 40,000 amps, rms symmetrical. The service entrance phase conductors are 4/0 copper installed in nonmetallic conduit. The distance between the transformer and the main panelboard is 200 feet, Determine the 3-phase fault current at the panelboard, 76 Chanter 8 Power Distrbution 3.13 3.14 B15 3.16 3.18 The ABCD constants of an overhead three-phase transi been calculated to be ion Tine have A=D=090+,0 B= 15+j130 C= 0 +j0.002 The Fine is energized at one end at a line-line voltage of 500 KV and open circuited at the other end, Determine the line-line voltage at the open end and the charging current at the sending end. A 100 hp squirrel cage induction motor is to be started from a 480 V bus supplied by a 250 kVA transformer with an impedance of 5%, Determine the percent voltage drop on the 480 V bus if the motor is started at full voltage. Assume that the starting current of the motor is 6 times the motor full load current, ‘A 3-phase overhead 12.47 k¥, 60 Ha distribution feeder has the phase conduc- tors configured as shown in Exhibit 3.15, The conductors are 266.8 kemil ACSR. Determine the inductive reactance of the circuit in ohms per mile. A Oo O ——+—-— Exhipit 3.15 ‘tan industrial plant itis planned to use 4.16 kV switchgear with a standard short cireuit interrupting rating of 250 MVA. The utility supply to the plant has been selected as. a 12 MVA 3-phase transformer. The plant will include on-site generation and synchronous motors, which will contribute 30 MVA to the short circuit interrupting duty. ‘Taking into account the manufacturing tolerance of £7.5% for trans- former impedance, determine the transformer impedance thal should be specified, The service entrance conductors for a three-phase supply to a facility have been selected as two 350 kemil copper conductors per phase, Determine the minimum size copper grounding conductor at the service entrance panel. Determine the required THW copper conductor and overcurrent protection device setting to supply a 50 kW, 208 V, 0.90 power factor continuous load. Ambient temperature is 30°C. Determine the required THW copper conductor size to supply a three-phase, 200 hp, 460 V squirrel cage induction motor, Ambient temperature is 30°C, 13.2 KV, 480 V 3-phase uransformer is equipped with fixed taps in the primary 13.2 kV windings. The tap is set at 5% below nominal. The actual voltage in the area where the transformer is installed. does not vary appre- ciably from 12.9 kV. The voltage regulation of the transformer is estimated to be 3%, Determine the low side voltage at full load. Sokeons 77 3.21 The equivalent sequence impedances at a 69 kV bus of a 3-phase power system have been ealeulated to be Z, (0.085 per unitand Z,= 70.20 per unit. The impedances are on a 50 M'VA 3-phase base and a line-line 69 KV voltage base. Determine the fault current in amps for a 3-phase, a phase- to-phase, and a phase-to-ground fault. 3.22 The equivalent system impedance of the supply to a 3-phase, 200 kvar, 480 ¥ capacitor is 0,005 + /0.030 ohms/phase. Determine the percent volt- age rise when the capacitor is switched on, 3.23. Determine the maximum trip setting of an inverse time circuit breaker to provide short circuit protection of a eireuit supplying a 40 hp, 200 V induction motor. 3.24 Deterntine the required trade size rigid steel conduit to carry four 40 THHN copper conductors and one #2 bare stranded copper equipment grounding conductor. 3.25. A 3-phasc, 480 V feeder has one S00 kemil THW copper conductor per phase installed in nonmetallic conduit, The feeder is 200 feet long and carries a load of 300.4 at 0.85 power factor lagging. Determine the percent voltage drop in the feeder. SOLUTIONS 34 jent circuit may be represented as Exhibit 3.La. ‘Sending end Receiving end \— —s, i t if 7 Exhibit 3.10 A simpler approximation (on a per phase basis) is Exhibit 3.1b, x \— A 5 —1, . cake ~~" Lon pincer std The complex power, 5,,, (which may also be expressed as the phasor voltage times the complex conjugate of the phasor current) on a per phase basis is 7 45+ J30) = 25+ j10 mega volt-amps. 738 (Chapter Power Distribution The current at the receiving end, thea, is (25+ j10)x 10° > 10° = (0.25+ j0.1)x LO" A/phase. 4 ‘The shut and series currents are then found. Fey, SVN where ¥,= j HSOX37TAIXI0") = 94.25% 10% ('72 storm 25)x10* = 9.425 Alphase yg thy = J9AIS + 250+ /100 = 250+ j109.4 Aiphase Mh And the voltage drop across the series branch may be found by first finding the series impedance. Z= R+ JX, = (SOXO.1+ j(SOX37IK2% 10") = 5+ /37.7 ohms/phase Vong = Agae% = (250 + j109.4 (5 + j37.7) = =2.874-+ j10.07 kV/phase ‘Therefore the input voltage is y, og HVe=-2aT44 1007+ 297.134 j10.07 KV and the sending end shunt current is VY =(97.13 + f10.07) j94.25> 10) 1949 + 9.1545 Alphase. Therefore the sending end current is Bag, Lg = 0.9494 79.154 + 2504 j109.4 = 250.9 + 118.55 A’phase. Then one may find the sending end complex power (again complex voltage times the complex conjugate of the current) as Complex Power = (97.13 + j10.07) x 10%(250.9 ~ 118.55) = 25.56 + j1.38 mega volt-ampsiphase. Soutions 78 And, of course, the total complex power is three times this value, 76.68 + #4.14 mega volt-amps. 3.2 a, Often self GMD of a bundled or composite conductor is called “geo- metric mean radius,” or GMR. Self GMD may be denoted as D,, This term includes the distances of a strand or conductor from all other strands within the same bundle plus the “distance of the strand from himself.” or the self GMR of the strand, In the present line configuration we have two strands per bundle, thus we have four distances: D,., Dy. Dy, and D,,.. The self GMR of a single strand is fess than the actual physieal radius (R x 0.7788). This reduced radius is the above given self GMD of 0,0403 fi. and is available from tables. Converting all distances into feet, we abtain forone bundle D,=D, = YD. xD, XD, * Dy = Dy = De Note: We extract the fourth roat as we have hour distances under the radical 4 (8) 2, = 0.0403) (3) = = 0.246 fh sfo.00366 b. ‘The mutual GMD of the line, or D., GMD values outside the bundle is the geometric mean of all mutual . between the three phases. D,, =D, = {03.55 *170x140, where ab = 15.5" and a’b’ = 15,5" 18 4 b= 15542 8x! = 170" ° 2 1B I 62155-2831 - 140° a’b=15.5 7n%a 14.0) = {57,180 = 15.5 1. Di = YUS.5x 2V x32.5x2! 921,400 = 31 ft. Dd, = YD, xD, xD, = YiS5%15.5%31.0 =19.5 ¢. ‘The inductive reactance in obnvmile or X, = 2x fx 10 x 0.741 L log “*. The 10” factor is needed to convert the inductance L, obtained in mb/mile to h/mile to finally yield ohm/mile, 19.) e = 0,377 x 0.74! = X, = 0.377 0,741 1 log: 0246 0.530 ohm/mile. Chapter 8 Powor Distrbution 33 The crux of the whole problem is the large portion of induction motors ‘The power factor of induction motors at rated load is typically from 0,700 0.90 with some groupings of motors resulting in even lower power factors. For a power factor of 0.8, a motor drawing 225 kVA of power will utilize only 180 kW. 190 kW KW = Real power 1O5AVAR VAR = Reactive power 225KVA AVA = Apparent power Exhibit 3.30 If the KVA drawn in this case were equal to the real power required (kW), a 20% reduction in current would result. The reduction in current with present load would reduce the voltage drop, thus improving the voltage at the load. Fluorescent lighting with capacitors usually has power factors from 0.95 to 0.97, therefore it is not practical to try to improve the power factor any higher. To improve the power factor with the existing loads, capacitors should be applied. They have the characteristic of a leading power factor whereas induction motors have a lagging power factor, By adding capac- itors, their leading kVAR cancels out the equivalent amount of lagging KVAR, as shown in Exhibit 3.3b. 180 kW 135 KVAR resuting. 195 KVAR of from.a group of ens capacitors Exhibn. 3.30 ‘The above power factor correction results in [80kVA = 180 KW or power factor = 1.0. Another method of improving power factor is ta add synchronous motors for the additional motor requirements. The synchronous motor acts like « capacitor, producing a leading power factor and leading kVAR. Normally capacitors are the most effective in reducing system costs when located near the devices with low power factor. Here we are primarily concerned with the feeder to the building, but if there are any large induction motors or a grouping of motors, it would minimize local branch circuit voltage drop as well as the feeder, if the capacitors were located near the source of the power factor (pf). ‘An economic study of the situation should be made. Data from recording pf meters and kW meters should be gathered from as many places as feasible on feeder and branch circuits. Then a comparison should be done of how many and where the capacitors should be installed. The installation of the synchronous motors vs, induction motors with capacitors should be evaluated, Solitons Bt With the additional load, the voltage drop in the existing feeder may be too much even with unity pf. Then consideration should be given to using a boost transformer, It may well be that a combination of boosting and capacitor and synchronous mater will be the most economical solution. Boost transformers are much less in cast than regular trans- formers as they are just an auto-transformer. Improving the power factor beyond a certain point increases the cest disproportionately to the gain obtained, thus all alternatives should be weighed in making the ultimate decision. Using a boost transformer alone may mean that at light tosd. an over voltage may result, which could be undesirable. Revised resultant KVA ow synchronous Lead Protila Diagram rotors Exhibit 9.36 34 AVA,,.. = 150 MVA (This value was selected (o be a practical base between the two given MVA values.) Kon = 69 and 12 respectively _ (kV 1000 _ (69¥ x 1000 vhs RVAL 130,000 23° x 1000 150,000 KVA,... _ 150,000 =31.8ohms Zine = 0.96 ohms Ices Spee = OOO 1,250 av, 3x69 ames KVAne 150,000 faett = Fy, Vive Oa Zrsepa = 2 eampu = Zee ps. aw = j0.075 von 20750 pau. Fn fete = 72.00 pa. 82 Chapter Power Distribution a. Three-phase fault (Exhibit 3.4b) Exhibit 3.45, Ey Zig + Zasaat + Zy = {0121 + 0.75 + 70.100 = 0.971 2g 3 Za = 72.00 J0.971x j2.00 = = j0.655 = jq.974+ j0.200"/” E 10 hug = ~/1.525 amps a= 7 Fess 525 amps Fou 3 phase at 12 KY = dea ps % Haase 2 = 1.525 x 7,230 = -j11,000 amps b. Single-phase fault The positive sequence impedance diagram is shown in Exhibit 3.4c Exhibit 3.46 The negative sequence impedance diagram is shown in Exhibit 3.44. "a Ze (0.855 Exhibit 3.40 The zero sequence impedance diagram is shown in Exhibit 3.4c. Onets 10.655 10.855 70.428 Exhibit 3.4f 69 KV system zero sequence fault currents are by AY transformer. Bay = §0.750 1 - Zapen as = 5 Zan assumed) = 1.000 Zant = 329 nun = 0...direetly connected neutrals thus; 10 ; fo 75565540.655+0428) /°8 pa. x Foe 1D KY = —J1.740 «7,230 = —j12.500 amps. 3.5 The voltage components are as follows; r+aV, +a°V,) = 312004 {0.5 + [0.866 ~j200) + (-0.5 = j0,866X-100-+ j200)] = 2200+. j1004-173.2+50- j100 + 86.644 173.2) (596.4 + 86.6) = 198.8 + j28.86 W, +a°¥, +aV) = }1200+ (AS— j0.866X—j200) +(-0.5 + j0.866X-100+ j200)] = (200 + f100 - 173.2 +50-— j100 - j86.6- 173.2) = (96.4 ~ j86.6)= —32.13— j28.86 B84 —Chapter3 Power Disinbution j200 = 100 + j200) star point and the supply neutral. 1988+ 28.86 20+ 10 =32.13- 1 ;: 198.8 + j28.86— 32.13 — j28,86) fart ha haa = ip. pg 1988 + 28BE— 32.13 728.86) (166,67) = ‘166,67 ej j10 = 1, = 6,67 — j3.33 amps in line a. To obtain the other (b,c) line currents, f, iy tal,s tly = [(—5— 0. 866X198.8+ 28.86) + (-5+ j0.866)-32.13- P8865; 10 = (-99.4 — f1 72.16 — f14.43 +24,99-4 16.65 — 27.82 + j14a3+ rans =(-33- 5200) 9° ~$.33~ j7.33.amps in line b. edly tal tly (0.5 + jO0.866X 198.8 + j28.86) P 1 + (O.5— jO.866"—-32.13— f28. 86159 jlo = (-99.4 + j172.16 + f14.43 — 24,994 16.65 + j27.92- jid43~24.99) 92510 n- =iI0 500: = (-133.33+ j200) = 1.334 10.67 amps in line e. asacheck: = 1,+1,4/, or O= 6.67 — 3.33-5.33~ j7.33-1.33+ 10.67. 3.6 a. The load as seen at the left of the Jast transformer is Z=F x1 + jx 1 = 16 +/16 2, ‘The impedance of the load and the transmission line as seen-on the right side of the first transformer is z (1 + 16) + (4 + 16) = 17 +20, ‘The impedance of the lead and transmissio (on the left side of the first transformer) is as seen by the generator Zs (OY x 17+ fA x 20 = 4.25 + J5.0 = 656.249.6° 2. (The magnitude of the current through the generator is 1 = VIZ = 2406.56 = 36.6 A.) b, Of the several solutions possible, one method is to define various base quantities along the system as follows: a. At the generator 240V= 1 pu b, For the transmission line, Vg = 240(2) = 480 V = I pu AVA.) = JO KVA = I pu ‘ pe [2 = (1 + §4/23.04) = 0.0434 + (0.1736 pul. ©. At the load, 4804 = 120 V = 1 pu OKVA = 1 pu 10,000/120 = 83.33 A= 1 pu =I pu (1+ j1V11.44 = 0.6944 + 70.6944 pu] The total impedance (as seen to the left of the first transformer) is, Zag = 2 + Z, = 0.0434 + J0,1736-+0,6944 + 70.6944 0.7378 + {0.868 = 1.139.249.6° pu which is equivalent 10 139% 5.76.289.6° = 6.56.2. (The magnitude of the per unit current is 1= 1,139 = 0.878 pu, and the actual current is [= 0.88 ¥ 41.66 = 36.6 A) BB Chapters Power Distribution, , Ifthe load is 2 + j2 then, by using u values, only Z, needs to be recomputed. Z,, = (2+ jBM.44 = 1389+ j1,389 pu 0434+ /0.1736+1.389+ /1389 1.432+ 1.563 pu The actual (otal impedance is Z=1A32 x 5.161 + j1.563 x 5.761 = 8.25 + /9.00 = 12.2297.5°0. 3.7 a. Power input to motors Generator capacity 830 kW 0.8 = 1036 kVA Current requirement 1036 x 1000 =272A 3x 2200 Exnipt 3.78, b. Power requirements Induct. motor input 600hp ,. 746 = 497 kW 0.9 1000 ork Induct, motor current __ 497 «1000 =163A * 2200 xO.8 Synch, motor input A00hp 746 _ 339 kw 0.9 — 1000 Solutions ‘Exhibit 3.7b Synch. motor current 3532x1000 =102A 3220x085 Ira = fF 005, + 1,c080,) +(F,sin®, + F,sin, = ¥(163%0.8 + 1020.85)" +(163% 06-102 0.53)" = 221211.5°Adagging) . i = to 17.6 cas 1.5?” 0.98 222.06sin1 1.5? = 44.3 A = 222.06 tte? ly —Frgay 682-4 3= 1189 A ve kvaR = 189 1000: vi = 151 KVARyphase 3.8 Initial power factor angle Initial reactive power Q New power factor angle 6” = cos! 0.95 = 18.19° New reactive power (0' = P x tan @’ = 20 x tan 18.19% = 6.57 Mvar Qo = 12 = 6.57 = 5.83 Mvar 080.85 = 31.79" x tan @= 20 x tan 31.79° = 12.4 Mvar a 8B Chapter 3 Power Distroution ‘The normal delta current = 200/V3 = 115.5 A ‘The capacitive reactance per phase X = 480/115.5 = 4.16.2 With one phase open 480 V is across n reactance of X in parallel with 2X XOX Xq= X42X J = 4802.77 UGX 416 = 2.77 TBA 3.10 Ay = cos"0.75 = 41.41" Qiy = 100 % sin 41.419 = 66.14 kvar To make the power factor unity Qxy = —Qry = -66.14 kvar oy = tan (-66. 14/150) = -23.8° Power faetor of the synchronous motor = cos 23.8° = 0.915 leading 3.11 Convert the delta Joad to an equivalent wye Zp = (60 + OV3 = 20 + j10.2 Equivalent circuit (Exhibit 3.11) 2y=20+ 100 Exhibit 3.11 _ $603 _ 266° = ape fig 7 N9E-266° A V, = 460/)3 + 11.9.2-26.6° « (0.50 + j0.75) = 275.21.11° ¥. The line-line sending end voltage = V3 x275=476 V. 480/32 312 The source impedance = 28043 Assume the source impedance is inductive. From NEC Table 9 the conductor impedance is. Z= (0.062 + J0.041) x 200/1000 = 0,0124 + j0,0082 Q 480%v3 0.00693 0.0124+ j0.0082 = 0.00693 2 167.2-50.7° A. Soa Soutions 88 313 V,; = AV, + Bly With f, = 0 (open circuity Vp = Vy/A = 5010.90 = 556 kV L=CV,+Dip Again with /, = 0 0,002 x 556/43 = 0.642 kA or 642.4 h=CVe 3.14 To take into account the motor power factor and efficiency it is reason- able to assume that the motor rated KVA = motor rated horsepower. Using a 250 KVA. base Serr = 100/250 = 0.40 p.u. jO.05 au, OD & jost7 estan) Vv, toa pu Exhibit 3.14 At rated valtaze Iraxy = 040 piu. x040=24 pu. /2.4 = 0.417 p.u. . joy ~ —~ = 0.89 p.t "70.4174 0.05) pa Voltage drop = |= 0.89 =0.[1 pa. or 11% Foon Xectod ee = 3.5, From a table of conductor characteristics the geometric mean radius = 0.0217 fi. D,, = Wax3x7 = 4.38 fi X,, = 0.1213 In (4.38/0.0217) = 0.644 Q/mile 3.16 ‘The maxinum fault contribution from the transformer is 250 ~ 30. = 220 MVA. ‘Using a 12 MVA base, the contribution = 220/12 = 18.33 p.u. Al rated voltage the transformer impedance cannot be less than 1118.33 = 0.0546 pu. To account for manufacturing tolerance X,... = 0.0546 + 0.075 X Xp. Xu 7 0.059 pu, or 5.9% 3.17 Cross-section area of the phase conductors is 2 x 350 = 700 kemil From NEC Table 250.66 the minimum size grounding conductor is 2/0 copper. 80 Chapter Power Distribution 3.18 3.19 3.20 3.22 Load current + 50,0004/3 x 208 x 0.9 = 154A Because the load is continuous, ampacity is based on 125% of this. P= 1.25 x 154 = 193A From NEC Table 310.16 select 3/0 copper THW 200 A, Select a 200 A overcurrent protection device. ith an ampacity of From NEC Table 430.150 motor full load current is 240 A For conductor sizing NEC requires 125% of this, 1.25 x 240 = 300 A From NEC Table: 310.16, select a 350 kemil THW copper conductor with an ampacity of 310A. Primary tap setting = 0.95 x 13.2 = 12.54 kV No load ratio = 12,540/480 = 26.125 For a primary voltage of 12.9 kV the no-load secondary voltage is 12,900/26.125 = 493.8 V. At full load the secondary voltage is 493.8/1.05 = 470 V. Iypuse = E12, = 100,085 = 11.76 pau. Toae™ 50,0003 x 69 = 418.4 A Fy ame = 418.4 X 11.76 = 4920 A Because Z, = Z; the phase-phase phase value. Frew grace = 0.87 % 4920 = 4280 A Taegu = JEMEZ, + 2,42.) = 30.085 + 0.085 + 0.20) = 8.11 p.u. Foro re = 418.4 8.11 = 3393. It current is 0.87 times the three Assume the capacitor is wye connected. 277 Xe = 550,0008 11510, Equivalent cireuit is shown in Exhibit 3.22. os WAAL (0.005 6 (0.0300 fc M Exhibit 3.22 on 5" _ 0,005 + 0.030 j1.151 ‘Voltage change = (284.4 ~ 277277 = 0.027 or 2.7% = 284.42-0.3° V Electronics OUTLINE PROBLEMS 93 SOLUTIONS 99 RECOMMENDED REFERENCES 111 PROBLEMS 4.1 A single-stage FET power amplifier needs to be analyzed so that parameter values and specifications may be established. In addition, the FET circuit may have an interchangeable op-amp as a preamplifier for various input signal conditions, ‘An n-channel junetion FET power amplifier circuit has the drain charac teristic curves shown in Exhibit 4.1. The FET circuit eventually will be driven by an op-amp acting as preamplifier for various signal sources. The FET has A= 100K0 Chapter 4 Bectronics a maximum drain voltage upper limit of 16 volts. Assume the operating quiescent current, f,, for the load line: is 22 mA and any operation is at mid- band frequencies, The following information is needed: Vs and R’, for maximum possible output signal excursion, b, ‘The transformer tums ratio n for an R,, = 8 02 for the R (as found in 1). ¢, ‘The approximate power output, graphically determined, when v,, 05 sinax volts. d. The percentage of second-harmonic distortion for jy for the same as in (c), e. With the op-amp connected and with a V, = 35 mV (rms), the value of R, needed to produce the signal in (e). ignal A transistor (BJT) amplifier is driven by a high input impedance MOSFET (p-chan, depletion mode) preamplifier. The tentative circuit design has already been proposed. However, a check of the circuit and performance specifications needs to be completed. ‘The component values for the circuit are given and some of the para- meters of the MOSFET and BIT are known. These are as indicated in Exhibit 4.2. Vos ¥ 1g ¥# Ugg curves. FET BT Sq = 25% 10° S(mho} 72400 4p, V5 Vgq.eurves (As shown.) 100 Vag =0.7'V Exhibit 4.22 ‘This problem is an adaptation of Problem $ as prevented io James W; Morrison's Principles andl Proctiee of Electrical Engineers Examination P&PIE, ARCO, 1977, pp. 113-117. 43. 44 Probiems 85 put impedance, and gains, Specifically, the following items are needed: a, The de operating point, Q for the FET. b. The de operating point, Q for the BIT. ¢. The small signal input impedance. 4. The voltage gain at mid-frequency. Assume the BJT transistor shown in the circuit in Exhibit 4.3 has a B= 100 and that og = 0.7 volts, +5 Exnibit 4.3 Find the following quanti a. To. dp, and Ve. b. Ba ¢. Drawing of the small signal equivalent cieeuit using a T model for the BJT and calculation of the gain, v,/v, rg and 4, A two-stage transistor (BJT) amplifier uses constant current sources for the bias portions of the circuit: the first stage has a fixed constant current source while the second is adjustable. The component values for the circuit are as given and some of the parameters of the BITs are known to be |Vg_/= 0.65 volts, B= 100 (since high accuracy is not expected, lis usually considered high), V, = 26 mY, and all capacitors are considered large unless otherwise indi« cated. The circuit is as shown in Exhibit 44a. ‘The tentative circuit design has already been proposed, However, a recheck of the circuit and its performance specifications needs to be completed, Assume the second current source (at V,} has already been adjusted and tested to produce a constant 2.0 mA. a. Determine the de bias voltages at V,. b. Determine the de bias voltages at V,, c. Determine the de bias voltages at V,, 96 = Chapter Electronics exhib 4.¢a 45 d. Find the input resistance R,,. e, Find the ac voltage gain from V, to V; and assume R, is disconnected. f, Determine the transconductance of the transistors Q, > g. Determine the output resistance at V, (include R,}. ‘Now assume the simplified constant current source is made up of matched transistors with very high 7s and is readjusted to give constant current source of 3.0 mA. Again, assume Vqy = 0.65 vols. h. If the original setting of Rs was R,. what should the new setting be (in terms of &)? i, Determine the numerical original value of R,, =R, (assume V, 4. Determine the de bins voltages at Vy. ‘Two different MOSFETs are being considered for use in two different circuits (Exhibit 4.5), Both transistors have a V, of 2 volts; however, each unit has a different K factor (conductivity parameter). +10 sav 7A THe Gp? Gyo s s 1k ska Ay Circuit 1 Circuit 2 =15 volts). Frobems 97 a. For circuit #1 the K factor is 0.25 mA/V? and you are to determine /,,. b. For circuit #2 the K factor is 1.0 mA/V? and you are to determine Ry for a drain current of 1 mA. 4.6 ABUTtransistor isbeing considered for use ina single-stage amplifier circuit As part of the analysis, with only limited information on the device, you arc asked to present an equivalent hybrid- mode! that will “fit” into a previously designed circuit (Exhibit 4.6), All that is known about the BJT is that B is 100, and that ¥, is 100 volts (Early voltage). osv = ‘Exhibit 46a Proviously designed ampiior circuit a. Draw the small signal equivalent circuit using the hybrid-# model and calculate &, and R, b. Calculate the overall gain v,fr,. being 4.7 A high gain op-amp having a unity gain bandwidth of f, = 1 MH considered in (wo different circuits as shown in Exhibit 4.7, i, 20ko Cireua ¥ Exhibit 4.7 a. For circuit #1 you are to determnine the voltage gain and the bandwidth. b, For circuit #2 you are to determine the input resistance, R,- 98 = Chapter Sectronics 48 A two-stage BJT amplifier has already been designed and is shown in the circuit in Exhibit 4.8a. Exhibit 4.80 49 41 ‘The A-parameters associated with this design are given as follows: Be (hy) = IK, Igy (ita) = 50, hag (hy,) = 2 10 * Foe (Roa) = 20 10°" mho Using an h-parameter model, you are to determine the overall mid-frequency gain, v,/i, Any assumptions made should be clearly stated. The cireuit shown in Exhibit 4.94 is an N-channel Junction Field-Effect ‘Transistor with self-bias and a pinch-off voltage of -3 volts. At that value of pinch-off voltage, the current is 6 mA. The breakdown voltage for this transistor is 30 vol Design the circuit in Exhibit 4.9a so that the device will be biased at approximately 10V drain-to-source and have a channel current of approxi- mately 4 mA. s A two-stage BJT has already been designed, but a complete analysis is needed for gain and losses. ‘Calculate the power gain of the two-stage amplifier shown in Exhibit 4.103. Show the gain of the two individual stages, the interstage losses, and the prestage losses. The ground-based h parameters for the transistors used in both stages are hy = 50 ohms; hi = 5X 105 tig =-0.97 and ity = 10 - 6 mho, Exhibit 4.10 Assume reactance to be neglaible SOLUTIONS Vowpe _ 16 44a Pe a Pedy v, “vy 6- (as referred 10 n side of tansformer) = Buermn) Ves _ 16-8 i, 23ma =3M0 b ReantR, ata Kin 455, nos R ©. A load line plot fora v,, of 0.5V on each side of the quiescent point yields: Vacant Vamp OF HLL an 4.2 volts ad Laps dM Fagg) OF 31 and 2 mA, Ai, _ (11-4.2)x (31-12)ma = 16.4 mW. 22 3 " ©. Vig) = 35 MV = 0.05 (O-to-peak), —_¥,, = 0.5(0-t0-peak) R . 1. Toy hey = et = ICV.) — Fefon) FRg, h, Changing the constant current source from 2 mA to 3 mA makes Rus se = QAR, i, fe=2 mA = [-C15) - O.65VR,, R, = 7.18 kQ J. Vy — Vy = 10 - 0.65 = 9.35 vats. 45 a. For circuit #1 the following relationships are obvious: Va = Vo = 1OV5 Vos = Vos ~ Vi Vos = Vast ¥os > Vos sa Solutions 103 5 = 1 kd = 10 don 1 = 0.25(10 = 1-2)", Ala = (B= Ip) = 64+ LE = 16lyy 1, = +1044 10064 = 10-6, Jy = 4 mA (the 16 mA is not acceptable). ‘b. For circuit #2 (with a K = 1 mA/V*) the calculations for R, for a drain current of I mA are as follows: Vi, = [12/(745)]5 = 5, 46 a 2ka a c Boa 9 3 5 Exhibit 4.65 R, and R, may be found by going through the following calculations: te & 1 mA (constant current source), HV, = 1 mA/0.0025 = 40 mALV, Big, = 10040 mA/V) = 2.5 kQ, V Hf. = 10/1 mA = 100 kQ. ‘Then = 10 ki, = 10 x 2.510 + 2.5) = 2 kQ. And for r, set, thenv,=0° and g,v,=0, therefore R, =, []100k, R, = (100 x 100)/(100 + 100) = 50 KO. b. The circuit gain may now be found as 8.1 (Fal 100k), Up = [flr + 2kQY]», = (22 + Pe, = 2, ¥ then the overall gain is vite, = = (2) g(r {1 00k) = = (1/2)40150) = —1000 104 Chapter 4 Blectrovics 47 a 48 Because the difference in the input voltages v, and v, nust be very small and the input current to the op-amp itself is negligible, then vy =¥, =O, y, = (RYAR)Y, yy = — (RAR, = -100/10 = -10, For the bandwidth, @, = (1 + RR, )a,, Li Sse = LE) + RalR,) = WK + 10) = 90.9 KHz. For circuit #2, the input resistance may be found as follows: R= vl. v=? 4, ¥4/10k, 10i = 0 = 10k(,/10k) = 20K = [xy — (- y)20k vel + vy ALOK = ¥/5k, (fv ISK) = 5 kD. ve w/10k. 4 I R, Using the f parameter madel will first find the voltage gain of stage 2, (vy and the input impedance looking into Q,. Then we can find the gain of stage I as loaded by stage 2 For stage 2, the equivalent circuit in Jt parameter form is shown in Exhibit 4.8b. Let 1 * he spyag 1.740010" mhos hee, =—fL=-] Sp o Tea MIO, (UD) 5 = higiy + Prev'y Then _ thgh a — tie tg tH, tL By thy hig - = 1000- (22 LO y= In +¥, 1000 — 119 10°(2« 10%) =976 Q. Solions 105 From (1) above, we have Malle #¥,) h vp(B.4 x10). Substituting into (II) gives vyon,[ tlt |g, 2 Ny = (1k)(84 x 10, — 2 «107 vy =-8.6 x 107 r, 0 16. Now we take an equivalent circuit for the input stage including the loading by the input to the second stage and the biasing networks. Hyg 350K] 30% 100K|20k Exhibit 4.86 We note the fact that the 2k collector load is in parallel with Z,,; (97602) and these are in parallel with the bias network for transistor #2 Note that 100k/20k = 16.7k while 2k/9760 = 65642. Therefore assume bias network for stage #2 can be neglected. ‘The unbypassed emitter resistor in stage #1 presents a very high input impedance to the source, (hy = hig tty + DR). S65 However, the h,, of the transistor is high compared to the SO0Q se we pn may assume that we can neglect f,, as being high resistance [1/h,. = 50k] 0.966 v, © compared to either the load (6569) or the S00 R,. Finally, we will noah incorporate the bias network for transistor #1 into the source by taking a Thevenin equivalent circuit By the previous discussion we will refer the &, (SOQ) resistor'to the input by Exhibit 4.8 Ry = (Me + 1) Rp = 25.5 kQ. 106 © Chapter 4 Seetrenies Now our equivalent circuit looks like Exhibit 4.8¢, _ 0.9640, -hivs 9654 h, + 25.5k =—2 “(656x503 0.964v,-2%1 “965+ 1k+25.5Kk - 838 x 10x, = 0.964v, — 2 x 10%, = ¥, (838 * 10) = 0.964 v5 ~itg | (656) 305% 10% y, = 965 My | Zaye, = 6560 25.5k a Exniit 4.8¢ Then the total gain 2 (21. 15-116) = 134, > Fs ‘The gain of this amplifier could be estimated teasonably as follows: Stage #1 has strong current-sevies feedback (the unbypassed emitter we Where R, is the 2k collector load Cin parallel with #,, of Q;) and the R,,_ is the Ry plus Jy. My is approximately iz = 200. resistor). Thus its gain is very nearly: ‘Thus stage #1 has an approximate gain of 28. icky, For stage #2 the gain is approximately —7~'* where My Ris Ski[Sk or 25KQ. Then Gain #2 ~—F5K150) _ 195, Tk Finally, the voltage divider of the input stage and the source resistance Zi, #1 by, + (agg + 1) Ry = Uk + (SISK) = 27k. Sotions 107 ‘The biasing network to stage #1 was found to be about 27k so that the combined parallel input impedance is about 13.5 k Then v(13.5k) = = 0.9305. ‘135k +1k Then the overall gain is estimated as follows: G == 0.93=125K=1.28) = 149. 5 This is about 115% higher than found by the more accurate procedure: well within the tolerance of the known parameters. 49 Fora junction FET Ios) : given that sazo{i-Mal) =| Vl 0.55 Volt. Gate leakage current fogs is typically of the order of nanoamperes. Choose R= | MQ, so that the gate remains within millivolts of ground potential, Choose R, to obtain the proper Ves. SS vols, 1,=4mA = R, =n =1409 Power supp of team ° ov 108 Chanter d Eisctronics 4.10 Because the breakdown voltage is 30 volts, choose Vip, = 24 volts. Vis = Vio — da (Ro + Rs) 10 = 24-4 x 10"? (Ry + Ry) 4 _asoon 4x107 If 10% resistors are used, the available values are Ry= 1500 and Ry =3.3k2. Solution is based on the following assumptions: Mid-band frequency—reactance of capacitors is negligible. Power gain defined as power delivered to load divided by power deliv- ered from source (this is power delivered to Rj, divided by power delivered of the vollage node at the junetion of C, and R,:). Power losses are defined as Pre-first stage—power lost in bias resistor Ry, Inter-stage—power lost in R,, and Rs Power losses—at signal frequency only, de bias losses not considered, Parameters are given in common base configuration; because éran- sistors are operated in common emitter orientation, the parameters must be converted to common emitter farm. Also, gain and impedance equa- tions must either be derived or found in a wansistor handbook. Handbook conversion tables (see Table 4.1 in Electrical Engineer- ing: License Review) give the following relationshi hy hgh, A, =—*—=16702 b= ad 4 eg ng LIOR eee tT 1 32h, 33 10° mho and Eyl. Ze 4 Eq. Ays"2= ee Big FR lige — Melty) MGs +g cg Ie) Exhibit 4.10¢ The gain of cach stage may be calculated by breaking the circuit as shown in Exhibit 4.10e, Definitions: v, = voltage at base of Q\vp = vollage across load Ry, A, in parallel with Z,,} "Rs (R,, in parallel with Z,) Iiage at collector of @, =-320 * (1670K0.5 10-*) + (16703.3 10°) —(11.7 104932) To calculate gain of first stage, the impedance Z,_ must first be obtained, This appears in parallel with R,, and R,,. all of which appears. as a parallel load for stage Q,. Then'Z,,, (from Eq. 1 is given as Ith ho+G, =1670-450= 1220.0 110 Chanter Bsctronics Then the effective load for Q, is et, 1, 1 21 SK” 10K "122K ~ 895 ‘Then (irom Eq. {1}, Meg = te Ma, AEG ehh) _— 3 (1670 }(1.12 x 10°) + (1670)(3.3 x 10°) (11.7 10“ X32) =-16.9. The total voltage gain from v, to vy is (-320)-16.9) = 5400. Tofind the power delivered by the source, Z,, must be calculated (Eq. D. ¥, high Va oh, — ig +G;, 1670-17101) (3.3% 10%)+ (112 x10") 1670-30 = 16402 Then the power delivered by the generator is a = Parallel combination of R,, and Z,,, and the power delivered to the load is ye yi R, 20x10" Then the power gain is = oasis =0,0625(5400)" lassie 82%10° 0 logG = 62.6 db. Recommended Aeterences 111 Now consider the inter-stage power loss (loss in parallel combination of R, and R,,): RMR . . CR, MR) (10% 10°X10%10") =333x10'0 Rit (10 10") + (5x 10") a By em * 333x107 ‘The total loss is then pa, 6.9%, vee $x10" 3.33% 10° ‘but v, 0.556 €, P 5560.7" (16.%0.536e, ))° a 5x10° 3.33x10° =27,7 10%? watts, RECOMMENDED REFERENCES Carlson and Gisser, Electrical Engineering Concepts and Applications, Addison- Wesley, 1990, p. 286. Savant, Roden, and Carpenter, Electronic Circuit Design, The Benjamin/Cummings Publishing Company, 1987. Sedra and Smith, Microelectronic Circuits,2" edition, Holt, Rinehart and Winston, 1987, pp. 350-51. CHAPTER ;— Control Systems PROBLEMS OUTLINE PROBLEMS 113 SOLUTIONS 119 Su 52 ‘The following system has already been designed; however, the performance parameters need to be specified. Determine whether the following system is stable and predict the closed loop pole location for the system for X= 4. Also, find the system error. Exhibit 5.ta The open foop transfer function for a control system is approximated by ots) KG-3) EG) (s+ 0.5)(8+7) It is desired to make the output signal (C) correspond as nearly as possible to some input signal, (2), in steady state, at the same time keeping the system stable, & Sketch a block diagram for a feedback control system to accomplish the given objective, Carefully label the summation polarity of all signals coming into the feedback junetion summing point. (Note that the given transfer function has peculiar properties.) b. Select a value of K which assures system stability and at the same time brings the ratio C/R in steady state as close to +1.0 as possible. (Note that the properties of G(s) are such that it is advisable to make a very careful check on the requirements for closed-loop system stability.) Gis) ns Determine the total effective polar moment of inertia, J.q. of the motor- load combination (as referred to the motor shaft when connected (a = 4) ideal gear train, Determine the uransfer function of the motor-load combination, Gu... ‘The transfer function (here, assume the power amplifier has a gain of unity, then the error voltage, E, equals amplifier control voltage. V,,..) is given as Gyn = OWVing, = OSE = Kits +1). . Show the system block diagram (ready for simulation) with the numer- ical values for the transfer function(s). For a unit step input of one radian applied to ©, and for a different power amplifier gain (other than unity), the error voltage as viewed on a strip-chart recorder is shown in Exhibit 5.6b, ‘Determine approximate percent overshoot and time-to-first peak for the closed loop system, From your calculations for part (d) do you expect the gain was greater ‘or less than unity? Why. ne Chapters. Contrel Systams For problems 5.7 through 5.16, the positional movement of a robot arm has a theoretical open loop transfer function of 57 58 59 5.1 $12 5.13 Gls) = ClsVE(s) = Ks + AW s(s + DNs + 2). If an optical sensor is used to monitor the arm position such that a closed loop system is achieved, what is the closest gain, X, that would just cause system instability (assume the electronic optical sensor unit is equivalent to unity feedback)? a O b 20 ce 40 d. 6.0 In open loop the arm transfer function has a frequency response such that the phase shift will be 180° at a frequency nearest to what value (in radisee}? a 0 b. Infinity «30 d 30 In closed loop (with the unity feedback sensor connected) the percentage overshoot of the output is to be near 16% for a step input. What would be the approximate damping ratio, zeta, (if the system may be approximated by a second ordered one)? a. Overdamped (nonexistent) b. 05 ce 07 d 10 In closed loop (with the unity feedback sensor connected) and with ait increased gain setting, it is found that the system will just break into oscillation. What is the closest natural frequency (in rad/scc) of this oscil- lation? a, Eventually reaches infinity b. LO c 30 d. 6.0 For the system in closed loop (through the optical sensor, for unity feedback) the gain has been set such that the damping ratio (zeta) is 0.5. What percent overshoot may be expected for a step input? (Assume the system is approx- imated by a second-ordered one.) a. 16% b, 25% 37% d, 50% For the system in closed loop (unity feedback) to have a damped natural frequency of 1.0 radi/sec, what is the clasest gain setting for K? a 6.0 b 40 ce 20 d 05 The arm (in closed loop) is to follow a moving object with a slew mate (ramp) of 2.0 rad/sec, What is the closest steady state error of the arm and the object for a system gain K ? a Zero b. 0.25 c 40 d. 10.0 8 Chapter §. Control Systems. determine the percent overshoot (to a unit step input) the system would have for this same value of K. If, for the complete system, a compensator consisting of a pure inte- gration (1/s) is inserted in the forward path (cither just ahead or just following the subsystem), what value of K would cause the system to be marginally stable? What is the natural frequency of this marginally stable system? 5.18 The system shown in Exhibit 5.18a is to be stabilized by the addition of tachometer feedback, K,. Exhibit 5.188 Find the minimum value of X, such that the system will just be stable. If the value of K, (found in Part (a)) were increased by a factor of 1,25 lie, 1.25 x X, min}, determine the approximate step response charac teristics. (Hint: one method of an approximate solution is to use only a reasonably accurate root-locus sketch and then approximate with “standardized” 2nd order curves.) Sctuwons 119 SOLUTIONS 5.1 For system stability use the Routh-Hurwitz method, G Rist) _Ktstb 1+G 24 2)+KG4) Pe Ds + Ks K G, Characteristic polynomial: s* +25? +Ks + Routhian array: so) kK ° 2K es whew y= QIU. (}an-0 yea EK Exhibit 5.1 . The first column is positive for all positive values of X and the system is stable. The root locus is sketched hy setting G = —1 (that is, us the basic rules of root locus, |G] = 1, @ =¢n180° with a being any odd integer) and one ablains a sketch like that in Exhibit 5.1. Exhibit 8.1¢ ‘The system is stable for all values of positives A°s. K 4 OS +IP + KS+K (st@lis+ ot iB with K=4: @5125, c=045, Bois G. 120 Chanter § Control Systems. 5.2 The system error is zero for both a step and ramp inpat, but is finite for an acceleration input (a/s"). ‘Therefore the error is 0.5 of the acceleration input The block diagram could be given as in Exhibit S.2a (with either positive or negative feedback), Kis-3) (e+ O.8NE +7) Exhibit 6.20 * For negative (-) feedback, the system function is, G K-39) I+G (s+ 0.5)s+7)+Kis-3) K(s-3) “FHG5+ Ke +G5-3K) Goya = The denominator (which determines the “character” of the response) must not have any negative factors (indicating closed loop poles in the right half plane, or an unstable system). A simple test for stability is the Routh criterion; or in this case (for a simple second-order system), all of the coefficients of the denominator polynomial must be positive. therefore, =F __Ks-3)_ Se 1G (54 0.5)(5 + 7)+ K(5—3) Here, for stability, again the denominator polynomial must be positive, thus 3Ks 3.5, K< 1.167. Of course the root locus method of analysis may also be used as shown in Exhibit 5.26, Solutions 421 b. For CIR to be as close to unity as possible, it is only necessary 10 minimize E = R — € (sinee it is a unity feedback system). be io s9lano plane + * 9 a ie 7 Rootlocus for oot locus for negative 1b. postive Lb. {angtes sum to £160") (angles sum to 0°) Exhibit 5.2b For negative feedback, R E . 1+G Now assume a step input (z = 2) and that one is interested in the error after a long period of time such that the final value theorem may be applied. ay Tose 1-0.857K 14 S05 To minimize this cquation, the term, | = 0.857K, should be as large as possible. For system stability the maximum: value that K can have is 1.167. If K is at maximum value then 0.857 K is equal to 1.0 and the term is zero. This causes ¢ to approach infinity. If K is zero then the transfer function, , goes to zero. Hence it is obvious that K must be between 1.167 and zero, If K were near zero, then there would be no forward path for the signal and the system would be useless, Referring to the left-hand s-plan root locus plot on the previous page, if K equals 1.167 then the close loop pole is at the origin and the system would act as an imegrator (meaning that the error would go to infinity after a long period of time}, Some value of K between the extremes would cause: the error to be greater than the input step. One may conclude that the real path for the root locus is between the -0.5 pole and the zero origin. This problem suggests that one must use some kind of compensation netwark, of, better, should review the original open loop transfer function for possible hardware changes such thai the negative zero perhaps could be made positive. An ideal location of the zero would be to the left of Soknons 125 Exhibit 5.40 The new forward transfer function then becomes: + Ks fo (5B) ‘The error again may be found using the final value theorem, To reduce the new error by a factor of 10, try increasing the stiffness, K, to give ‘The new root locus will then have a zero Located at ~1/Kp (yet to be determined). Because the amount of dampi to remain the same (ie., the “Q" line to remain the same), the locus will be of the form shown in Exhibit 54e, The requirement of error, being Kye =10Keypaux and locating the new closed loop pole of the “” line will give the relationship of (10K) = K 106 /. 1268 Chapters Control Systems 55 Using this relationship, one may, by trial and error techniques, find the value of Kp. However, a more straightforward technique not using root locus methods will allow us to approximate K, directly, (This will be approx- imate as the zero in the closed loop response will alter the damping.) Consider, (OKXI+ Ks) _ oh +Kys) S'+ Boe Kp(IOK)s+(10K) f+ sSa,s+ a? But the new “{” and the original “{” are required to be the same, 9p BH1OKR giving: io 2 and “C” = 0.707, As an example, let K = 2+(10N2)K, Then 2= 2tHON2K | siving Kp = 0.216. Or, the zero (for the root locus plot) is located approximately at -1/Kp. which gives 4,6 for the numbers previously used. ‘Therefore the new stiffness should be 10 times the original and the “time constant” of the derivative compensator should equal Ky, Assume an ideal controller with torque directly proportional to error, E, (T= KE), and that the system is of 2nd order (Exhibit 5.52). T=KE=K(R-C) T=Us +899 Solutions 12 Time (zee) The percent overshoot is over 50% and ¢, is approximately six seconds. €. From the ofiginal block diagram (with unity gain) on¢ may easily obtain the closed loop transfer function as O/B, = 1G.25? + Is + 1) = 0.3125" + 0.3125 + 0.312). Here, , = V0.312 = 0,559, £ = 0.279 (well unde damped), From standardized second ordered curves, a zeta of 0.279 gives approximately 40% of overshoot. The percentage overshoot of approximately 50% for part (d) com- pared with the 40% (using the original unity gain setting)is 10% higher, therefore the new gain of part (d} must have been increased slightly. 3.7) d. Use the Routh-Hurwitz method. G K(s+4) SEG oh 3st (2K AK AERIS K og 3 2 6-K>0 K<6 ee 5.8 ¢, Use the Bode phase approximation method; sketch on semi-log paper, or use the direct calculation method with two or three “guessed” trial frequencies: Ais+4y | 2K(0.255+) s(s41)s+2) 584105841) ssija 2 = tan 0.25e - 90° — tan! @ —tan' 0.5@ = —180° 130 Chapter 5 Control Systems 59 ob 5.10 c. 5.1L a, 5.12 d. Try w= 2: 20 = 14 — 90° ~ 63.4% — 26.6° = -166° # 180° Try w= 3: 26 = 36.9" — 90° ~ 71.6° - 563° = -181° = - 180° Because the system is approximated by a second-ordered one, the normalized, standardized curves give a family of plots that relate zeta to percent overshoot. Or, the damping ratio may be calculated directly. . O16 =e" In 0.16 =-1.833 = bx a." = F205 From the root locus plot, find the frequency where the curve just passes through the imaginary axis; or, the value may be calculated directly from the closed loop characteristic polynomial (CP) equation. C.Pee+38+2+K)r+4K=0, for s=0+jar(on axis) = jaa? = 307 + joo? + K) + 4K (-3aF + 4K) + jo-oF + 2+ =0+0=0 n@=8 a= /B~3 ‘This is a “standand™ graphical relationship that is given in almost any text on control systems, and is plotted for an ideal second-order curve. Or it may be calculated from looe fF noe ** = 16.3 = 16%. OS. Carefully sketch the root locus in the region near 1,0 rad/sec (at the imersection of the crude root locus and a straight line passing though the imaginary axis at 1,0). ‘Once the correct focus is determined, X is the product of the line lengths from the open loop poles divided by the Tine length of the open loop zero. _ Lebel 2x18 3.8 6 5.13 b. 5.14 b 5.15 & 5.16 a. Soltions = 131 Exhibit 5.128 Because this is a type [ system (i,e,, one integration in the loop equation), the error will be finite and is found from the final value theorem, eo) Because the input is an acceleration (A(s) = 1s"), the looy ‘only one integration, and the error will have to approach infinity. This is somewhat lengthy to solve, One method is to use the original root locus to locate the original uncompensated zeta for a damped frequency of 1.0 (ie., a horizontal line passing through the imaginary axis at 1.0 and the root locus. Using this zeta line, locate the desired ew locus location on the zeus line and a horizontal line passing through the imaginary axis at 1.5. This new locus location will be incorrect by certain angle (c,, all correct locus locations should add to +/—n180°); find the difference between this certain angle and that of -180° (this difference will be approximately 35°), Then, by trial and error, locate the compensator pole on the real axis such that the new locus angles will sum to -180° (a location of 2.6 for this pole will satisfy the angle relationship). Generally the A matrix may take several forms, however here itis gi ‘The C matrix relating the output to the state variables may be found directly as follows: Ko+4) Gu) = 8S O° eaten i 132 Chanter 5 Control Systems 5.7 a. 4i, = 0x, +x, +0x, 4, =0x, +0x, + Lx, i, =0x,-2x,-3x,+le 0-2-3 I y=c=4K+Kx,+0x, ya[4k K 0) x= The subsystem is an ideal second-order system with poles located at —1 + jl and — j -J1. Obviously, because the poles are located in the left hand plane of the s-plane, it is.a stable system. Also, because the poles are fixed, any positive real value of K would result in a stable system. For the complete system the loci of the roots would emanate from the subsystem poles by going straight up and straight down. A zeta line radiating for the origin of the s-plane at an angle of cos! 0.4 = 66.4° (from the real 180” axis) would intersect the root locus at ~1 + j2.33. Because K is the product of the fine lengths from the open loop poles to the closed loop pole, it is found to be Kt, xt, =133K333=4.43, Again, because the new system is also an ideal second-ordered one, the percent overshoot may be calculated (or found from any standard set of second-order curves for a particular zeta) as follows: Percent overshoot = 100e""""" = 25.6% The forward path equation is merely Ki[s(s° + 2s + 2)], and since H = 1, the characteristic equation of the new system is 42s +254 K, A Rough-Hurwitz array yields where X= (2 x 2-1 x Ky2; for a marginally stable system X = 0, therefore K = 4, Solutions 133 Another way of finding the value of & for a marginally stable system is to allow sje fic, the root locus crosses the je axis for this condi- tion), then substitute joo for + in the characteristic system equation. (This method also determines the natural frequency.) Gio)" + jay + 2ja+ K = 0 = (-20F + K) + ja? - a) since both (-207 + K)= 0 and jo (2-@*) =0, therefore: jw=0, or w=¥2 rad/second and, K=lo'=4, 5.18 a. For stability use Routh-Hurwitz criterion for the characteristic polyno- mial: G,.<=__40 SR 5765410) +0 + Ks 400 Because the equivalent block diagram may be given as Exhibit 5.18b, Exhibit 5.18 Characteristic polynomial = 5° + 10s? + 400K,s + 400 ‘Routh-Hurwite Array: i 400K, e 0 400 ody (104004, )— (1,400) ere 1K, >01 for x,>0 (K, minimum = 0,1) b. Plot roet-locus for HG: let A = 1 + Ks = 140.1259 = 0.1258 + 5) 125x400Ks+8) | SO(s +8) s(5+10) #(s+10) Here K(root — locus) = 50. “HG= Because the solution is only approximate, let ¢, =f, 134 (Chapter 5. Control Systems Exnibit 6.186 and. Then {line = 80° and, = 50. f= 017 From any standard 2-order transient response curves, one can easily: determine percent overshoot and time to first peak, ¢,. GOS. = 60% 3.2m, = 0.45 seconds, en Exhibit 5.184 1396 Chapter 6 Computing 62 A certain requirement for a magnetic circuit (an iron toroidal ring With an 63 64 air gap cut into it) bas a unique shape for the cross section of the iron; it is triangular in shape, Obviously the iron faces on each side of the air gap are triangular and the area of the air gap is triangular (neglecting any fringing effect), One of the design requirements is to find the amount of magnetic flux in the air gup (same as in the ring) from certain required flux densities, B, and various triangular dimensions, (Recall, the flux equation is: @=BXxA,) ‘The lengths of the sides of a triangle are given by the values of the variables X, ¥, and Z. Then the area of the triangle can be computed from N+¥+Z AREA = JiV(W—XXW— XM 3 Z) where W = Write a BASIC, Fortran, C, or Java computer program to do the following: a. Input values of X. ¥ and 2 (in meters) and B (in Telsa) from the keyboard. b. Compute the area of the triangle and the flux. c. Output a heading identifying X, ¥ Zand FLUX, followed by their values in an exponential format. A tentative design for a control system to position a radar antenna does not inchide feedback compensation (such that minimal cost and weight factors may be achieved). However, if need be, feedback compensation could be added in by a value of X;, Before the system is built, it is decided to test, the design by computer simulation to check the performance of the system without any feedback compensation (i,¢,, A; = 0). The tentative design block diagram is shown in Exhibit 6.3a, Shaft output Exhibit 6.34 Of the many high-level simulation languages available, it is decided to use he default method of integration. Also, because the programmer jar with optimization techniques of analysis For ihe feedback compensation, the solution for the output vs, time for a step input will be programmed for Ky equal to zero; then if the results are not satisfactory, K, will be set in and incremented until the desired results are achieved, An ac circuit problem (Exhibit 6.4) is to be solved for two specific condi- tions, thus a computer solution is appropriate. Proviems 137 @ sr 008 secon 0001) Exhibit 6.4 Using any popular packaged circuit program, state the name of the program you choose and write oul your program(s) including any remark statements that would be suitable, Here, assume another person will actually nun the Program to obtain the solution and thus your remark statements will be needed for clarification, Your program(s) will be to find the steady-state ‘voltage and phase angle (with respect to current source) across the capacitor for two conditions: a. Fora fixed current source af s(t) = 2cos 1000r. b. Repeat (1) except the frequency will vary from 500 to 2000 rad/see in steps of 250 rad/sec. 65 A thermocouple is a sensor used for measuring temperature, It is made by joining two wires made from dissimilar materials. To measure temperature, two thermocouples are connected to a circuit as shown in Exhibit 6, One thermocouple is placed in a medium with a known constant tem- perature ,,.[i., iced water], and the other is placed where the temperature Tis to be measured. A voltage ¥ is generated when the two temperatures Ve Reterance Junction Ve Mout To the signal conditioning V ‘Themmecoupla wires and thee unetion Exhibit 6.5 138 Chapter6 Computing 6.6 67 are not the same. The voltage ¥ can be modeled as a function of temperature by an expression of the form Ve KT - Ty) where K is a constant that depends on the two materials that are utilized for the thermocouple. The following are the results from an experiment that ‘was done for determining the constant X's, of a certain thermocouple, Using this data, determine X's using Matlab or a similar program, and curve fitting. Tin degrees C 25 100 200300400 S00 600700 vin voltage 1 403 816 1262 1654 209 237 29.17 A communication system inserts extra bits into the data transmitted by atransmission stream, The extra bits must be removed in the receiver. Conversely, the receiver cireuit portion of the digitial communication system is required to remove the excess bits packed into the serial data stream at the transmitter. Using any commercial computing package, design the receiver fsm and control logic that extracts the extra bits that were inserted at the transmitter. ‘The resulting data stream must be an NRS [non return to zero] data stream that is a perfect replica of the transmitted stream prior to adding the required transmission bits. Describe the logic gates that make up the finite: state machine. A thermistor is used to measure the temperature of a motor from the lowest expected temperature (10 degrees Celsius) to the highest motor operating temperature (150 degrees C). Data for the thermistor is as follows: ‘Temp (degrees C) (ohms) 10 12.0 kohras 00 7.4 keluns too 4.5 kotums 00 2.8 kohms 300 1.8 kolums 40.0 1.2 kaens 500 1860 ohms 0 S60 70 400 800 230 soo 2 100.0 210 150 1iee 15 120 90 130.0 65 1400 50 130.0 0 SOLUTIONS 68 69 6.10 41 Souwons 139 The temperature isto be displayed on a voltmeter, from 0)V to 5 ¥. Describe the characteristics and components of a suitable electronic interface circuit, Assume you have general-purpose commonly available operational amipli- fiers available for Vec = +f-15V. Use only the standard resistance values for resistors with the standard industrial tolerance. A black box containing a linear circuit has an on-off switch and a pair of external terminals, The voltage between the open-circuited external termi- nals is measured at 12 u(t) Volts when the switch is turned on. The short- circuit current is observed to be [0.2e'") u(t) Amps when the switch is again tured on a. Find the current the box would deliver to a SO-ohm resistive load. b, Find the interface voltage across the terminals. cc. What is the power dissipation in temas of watts? A BIT transistor is operating in the active mode. The soutce is 24 volts and the resistors are standard values. Utilizing SPICE and the appropriate model, find the power delivered by the power source and the power absorbed by the transistor and the resistors. An induction motor is undergoing the blocked-rotor test. The motor will have a stated line current in amps, which is drawn when the line voltage is given in volts and the total wattage will be specified in watts. For example, the typical formula for wattage is volts x amps. Describe the formulas you would use in an interactive computer program that a technician could utilize 10 calculate a. the equivalent resistance reflected to the stator per phase b. the equivalent impedance per phase ¢. the equivalent inductive reactance per phase. Ifthe ohms per phase forac resistance of the stator are specified, the program should calculate d._ the resistance of the rotor per phase reflected to the stator e. the inductive reactance of the rotor under blocked rotor conditions. Because the system has differentiating block, sK,,(which is very difficult to simulate by both analog and digital methods), the block diagram is rearranged so that this block is ahead of the integrator as shown in Exhibit 6.1b. | aret Exhibit 6.16 140 Chapters Computing Exhibit 6.16 a. The analog computer equivalent schematic (Exhibit 6.1c) is arranged to have a pick-off point forthe system error al e, and the various gain controls may be easily manipulated for several different settings. Also, because all op-amps operate in the inverted mode and all loops are for negative feedback, all loops are to have an odd number of op-amps in a loop (this, of course, is not a requirement in a digital simulation program). Also, no numerical values are specified, so no provisions are made for magnitude or time scaling—which, again, is not a factor in digital simulation. b, No special circuitry is needed to obtain a step response; however, for aramp input, an integrator block (Exhibit 6.1d) is placed ahead of the summing block, a Exhibit 6.1d ¢. Most packaged digital simulation programs use an automatic variable step size routine for the actual integration, however, some of the earlier ones required the programmer to enter this value. As a rule of thumb this value of delta time would be less than. one-tenth of the shortest time constant in the system. (in BASIC) a&b. 10 INPUT “INPUT X,¥,Z IN METERS, SEPARATED BY COMMAS”, XYZ 15 INPUT “INPUT FLUX DENSITY ,B (IN TESLA)", B 20W=(X+¥+Z)2, 30 AREA = (W*(W — X)*(W - Y)"(W - Z).5 36 FLUX = BYAREA 40 PRINT “SIDE X",“SIDE Y".“SIDE 2" “AREA","FLUX" 50 PRINT USING “#.###°":X,Y,2,AREA FLUX 60 END ce Note: A typical output for a test input of X = 0.002, ¥ = 0.00282, Z = 0,002, and # = 1,0 (this portion, of course, would not be part of the exam) gives RUN INPUT X,¥,Z IN METERS, SEPARATED BY COMMAS 0.002, 0.00282, 0.002 INPUT FLUX DENSITY, B (IN TESLA) 1.0 SIDE X SIDEY SIDE W AREA FLUX 0.200E-02 —0.282E-02 0.200E-02 (0.200E-05 0.200E-05 Y= output 63 Because of the possible use of the derivative function used for the feedback compensation, the block diagram is first rearranged to avoid this operation, as shown in Exhibit 6:3b, e | ween eal +2) Exhibit 6.36 Or, for programming conceptuatization, the diagram is again modified as Exhibit 6.3c. It should be noted from a CSMP manual that a lead-lag network is given by VisVX(s) = (Pie + DCPs + 1). or Pudi) + y = P\(dutdi) + a. Then dts + Ivis +2) = 26 + IMO.Ss + 1). Thus, one possible simulation program could be 142 Chapter 8 Camputing 64 LABEL SIMULATION OF AN ANTENNA POSITION CONTROL, SYSTEM INITIAL CONSTANT K = 2.0, KT = 0.0, PL DYNAMIC INPUT = STEP(0.0), E = INPUT - Y — KT * YIDOT, El = K * E Y2DOT = LEADLAG(P1,P2,E1) YIDOT = INTGRL(0.0,Y2D0T) Y = INTGRL(0.0,Y 1DOT) OUTPUT =Y TERMINAL, TIMER FINTIME = 5.0, PRTDEL = 0.05, PRTPLT OUTPUT END STOP END JOB 0, P2=05 The computer will then print the output for these conditions and, if the results are satisfactory, the problem is solved; if, however the results are poor, different values of KT may be tried. One way of obtaining the step results for these values of added feedback compensation is simply 10 change KT'=0 to several other increasing values and repeat the operation. A better way would be to initially delete XT from the CONSTANT line, then follow with another statement line such as PARAMETER KT = (0.0, 0.2, 0.4, 0.6, 0.8, 1.0) Still another way would be to do this with a FORTRAN statement along with a NOSORT notation. The FORTRAN statement could be based on a conditional requirement of some parameter of the output response and then looped until that requirement is achieved; this method depends on the sophistication of the programmer with regard to both FORTRAN and knowledge of control system optimal requirements. One very popular digital package circuit program is PSPICE, which will be used here for demonstration, Fixed parameters CIRCUIT PROBLEM, ac_prob.cir ti 1 AC 2a Odeg Vmir 12 0 * Vinur is a dummy voltage source of 0 volts that reads current. HI 30 Vmir 5 RI 10 100hm R2 a4 4ohm ® As acheck for XL and XC recover from w = 1000 ws. Solutions 151 Then simplify by use of Kamaugh map. For f, the Karnaugh map is Exhibit 7.18 For f, the truth table is: J—|—|>: o 2 al-|- J, = AB+ BC+ AC (Exhibit 7.1b) ol» nt? Exhibit 7.1b One could have simplified the Karnaugh map in terms af “0's” as fo = BC+AB+AC 2 fy= BC+ AB+AC =(B+C)(A+C)-(A+ B) 0 go directly to NOR logic: 162 © Chapter7 Degtal System o> =p oie and f, = ABC+ABC J, = ABC+ABC = (A+ B4+C)-(A+ B4C) ¢. Exhibit 7.1d shows the final logic cireuit diagram for the alarm circuit. Of &) >i vy © c Exhibit 7.10 7.2 a. Exhibit 7.2b shows the interconnections necessary for the equivalent 1K x 4 bit RAMS (from the 1K x | bit RAMS). Solutons 153 b. Exhibit 7.2c shows the interconnections forthe 4 full adders, the RAMs, and the $ D-type flip flops. Exhibit 7.2¢ To correct the outputs of the natural binary signals to that of binary coded decimal, it must be recalled that natural binary includes numbers between 0 and 15, while BCD numbers are limited to 0 through 9. As Jong as the sum does not exceed 9 the outputs are correct. However, if the sum exceeds 9, then 6 must be added to the natural binary sum to yield two BCD numbers. As an example, suppose 7 and 5 are added together, the natural binary sum is (J 100}, while if we add 6 more, the new sum is (0001 0010}gcp- TA onl io oi01 1100 O10 ‘Therefore add 6. 120001 0010 Answer in BCD. ‘Thus it is necessary to detect if outputs of S's of the full adder exceed 9, if so add 6 with the additional circuitry. This detection is casily accomplished if the 8 line is high AND either the 4 OR 2 fine is high (and, of course, OR if the cy output of the MSB FA is high). ‘The two T-segment driveridecoders are connected only to show a 0 or a I for the MSB and a > 9 for the LSB. Exhibit 7.2d shows the interconnections necessary for correcting the full adders to correctly read the BCD sums and the connections to drivers, and the LED displays. 154 7.3 b. Use the truth table to form the function on two or more 1's for the sensors, 3 > s a Foon = ABC + ABC+ ABC + ABC. -se0--50 -e-c-c-o -- re -ccely 14 a 1S c 16 b 17 be. 18 a 19 «, 7.10 d. Td 7.12 b. Soutions 155 Form a Kamaugh map (Exhibit 7.4) for f= 22(3,5,6,7) f=AB+ AC + BC From the sume type of truth table as Problem 7.4, form the function when A, B, and C agree (i.e., terms @ and 7), Exhibit 7.5 shows the Kamaugh map. flactivate 2nd alarm) ‘lactivate 2nd alarm’ = ffactivate 2nd alarm) = (A +CXB+C\A +B) V=2.9=5(1—eW**) = 5(1-e*), x= 0.867 0.867 = (RCM, for R=SK,C 5 ufd 64 counts must occur in SO ms. 64/50ms = 1.28 x 10°, clock = 1.28 kHz Sketching a timing diagram will show that the added counter ouput should always start hi, otherwise the warning device will stay hi Note that (255), is for full scale for an 8-bit converter, Mid range corresponds to (127),, Which, in tum, corresponds 10 (7F),,, ‘The MSB corresponds to 128 and the next MSB is 64, and 128 + 64 = 192, therefore (192/255)5.0 = 3.75 volts. For the configuration of question 8 the “weight” af the two lines are equivalent to (192),, or 3.75 volts, therefore 3.90 = 3.75 = 0.15 volts, or (0.15/5.0)255 = 7.65 (nearest integer value = 8). Therefore the line that represents 2° is correct. Solutions 157 The two four-bit solution possibility is 0000 0000 0000 0001 0000 0011 0000 0111 0000 1111 ooo THT oon OM TL MAMIE LLL LL 1111 1100 i111 1000 1111 0000 1110 0000: 1100 0000 1000 0000 0000 0000 ‘The JK flip flop characteristic equation is Q* = JQ’ + K°Q. and the flip flop equations follow from this. If one utilizes the D flip flop for VHDL, you can use the Q*’S or next state values directly. ‘transmission line, length £, of 25 em and whose characteristic impedance, Z,., is 500. The frequency of the generator is 300MHz and the phase velocity, u, on the line is 300 x 10° mis. 8.2. The capacitance per unit Iength of the line is a 66.7 uF b 334 pF © 50.3 uF d. 66.7 pF A, = 750 Transmission tine 2, = 508 Exhibit 8.2 8.3. The inductance per unit length of the line is a 250-BH b. 166.7 HH ©. 25 mH 166.7 nH 84° The attenuation constant of the line is a LO b. infinity 2 0 a. 50 8.5 The phase constant of the line is a 6rad/m b. 2nrad/m ce. 1a rad/m a 4x rad/m 8.6 The input impedance, Z,, looking toward the load is a 250 b. 2500 e258 a. 0a 8.7 The input voltage at Z,, is a 280 b Vv «. 50V d. 100 8.8 The reflection coefficient, I. is a 0 b 3 eo W3 da 6 Problems 181 8.9 The standing wave ratio, SWR, is aol bh 1 a 4 d 2 8.10 ‘The load voltage, ¥,. is a -37.5290° V b. 5260° V ce SZ90°v a. S0Z90° V 8.11 The average power delivered to the load is, a 0S Ww be 125 w e. 125w a. 25w 8.12 A section of nonloaded telephone cable, located in a rural area, has the following characteristics per loop mite at a frequency of 1000 Hz: # Series resistance, 85.8 ohms * Inductance, 1.00 millihenry * Capacitance, 0.062 microfarad * Shunt conductance, 1.50 micromho. ‘The cable consists of 400 pairs of No. 19 AWG, is shielded, and is 3.06"0.d. Compute the following parameters for this cable at 1000 Hz: a. characteristic impedance b. attenuation in decibels per mile c. phase shift per mile d, velocity of propagation. 8.13 Exhibit 8.13a shows a diagram of a coaxial transmission line in which the circuit elements have the values inclicated, and E, is the open circuit zenerator voltage, Z is the characteristic impedance of all transmission lines, and £5 is greater than one wavelength, ee Z9 1204/60 Shorted stub Exnibh 8.138 Chapter 8 Communication Systems Miers tt tts i. Determine the length L,. in terms of a distance that is greater than one wavelength, and is the proper distance so that a shorted stub attached. at this point causes the line to be matched to Z,. Detei the length of the shorted stub L, needed to match the load. to the fine at this point, . Determine the VSWR on the unmatched portion of the line Ly. 1. Determine the power that will be supplied to the load under the matched conditions. . Determine the greatest voltage that will appear across the transmission line in the unmatched” section £,. “Note: 8 Smith chart és included to aid in the solution, or a straight analytical solution may be employed. “Smith” is a registered trademark of the Anulog Instruments Co, and is reproduced by courtesy of Analog Instruments Co., Box 808, New Providence, NH 07974. Problems 163 8.14. Two microwave stations operating at 5 GHz are 50 kilometers apart. Each has an amenna whose gain is $0 dB. greater than isotropic. If 6 wats is applied to the input of the transmitting antenna, what is the signal level at the output terminal of the receiving antenna under free- space conditions? What is the path loss? 8.15. A phased array consists of two half-wave antennas located a half-wave apart, as shown in Exhibit 8.15. Antenna currents J, and Jy are equal, and 1, leads [, by 90°, Maximum field strength of each individually excited antenna is 250 mv/m at a distance of 40 kM. Radiation resistance of each antenna is 73.19 (the same as a half-dipole). a Exhibit 8.15 Determine the angle, @, at which the resultant field strength is maxi- mum, and calculate the field strength of the array for @'= 90° at a distance of 25 kM. 8.16. In the modulating circuit shown in Exhibit 8.16a, the modulating signal (see fo) Spectrum of modulation signal Exhibit 8.164 164 Chapter 8 Communication Systems 8.17 8.18 8.19 spectrum sketeh) is limited to angular frequencies «a where 0, << 0, <9, and @, = modulating frequency; and a. Sketch the spectrum of the output. b. What is this type of modulated signal called? c. Sketch another circuit that would produce the same output spectrum, = carrier frequency. A certain transmitter has an effective radiated power of 9 kW with the carrier unmodulated and 10.125 kW when the carrier is modulated by sinusoidal signal. a. Determine the percent modulation at 10.125 kW output, b. Determine the total effective radiated power if in addition to the sinu- soidal signal, the carrier is simultaneously modulated 40% by an audio wave. In order to specify the communications link for a closed circuit television system, the bit rate must be known. ‘The monochrome television picture signal of this system requires 10 distinct Icvels of brightness for good resolution. This television system also includes the following parameters: (1) Frame rate, 15 frames per second (2). Lines per frame, 1200 (3) Discrete picture elements, 100 per line. Determine the channel capacity in bits per second required (0 transmit the above signal with all levels equally probable and with all elements assumed to vary independently, List any assumptions that you make. A series of remote stations is being planned to feed data to a large computer, ‘The data are to be sent by the remote siations and recorded on a tape recording unit and then fed to the computer as necessary, The data are expected to arrive from the remote stations with a Poisson distribution at an average rate of 10 transmissions from remote stations per hour, The recording time of the data varies exponentially, with a mean time of four minutes, a, What is the average waiting time for a remote station before the data will begin to record? b. A second tape unit including automatic switching equipment is avail- able at a cost of $2.50 per hour. The telephone lines cost 4 cents a minute per line when used. Is the second unit economically warranted? Show sufficient calculations to justify your answer. SOLUTIONS 83 84 85 8.6 Soutors 185 2 10° = 20187 ay 10 log, 240 = 10 x 2.38 = 23.8 db b. Loss in cable: 0.4 nepersémile x 2.5 miles = 1 neper 1 neper x 8.686 db/nepers = 8.686 dom P,, level = 0 = 8.686 = 28.686 dbm Pa level = 10 log = 10 log? x 10' = 10 x 3,3 = 33 dbm 110% db Gain, = 33.0 ~ (8.686) = 41.686 db ‘Source Ampidier O--> > -17-7& Eom apm 8.886 dom 3. dm Exhibit 8.1b 0.1255 radini_ 2.5 mi 10" microsecise: _ 54 icraseconds, Del: Delay 2x 1000 He d. The Toss at 1000 Hz could be reduced the least expensively by loading (series) the telephone line: inserting inductance coils at regularly spaced intervals. The delay of the transmission line will increase when load coils are added, because we increased the phase shift. a@ ZsVlC: matViC; Zm= ie. 1Z,48) = 1150 3 * 10°} = 0.06667 x 10” = 66.67 pF. dC = 1; b= 25000 x 0.06667 x 10" = 166.67 nH. c. a =0 (since the line is lossless), b B=a@VvLC =evu; a@=Inty B= (ex 300x 10°V(3 x 10") = 2a radin, a A= Cif=(3 x 10°300 x 10°) = 1m; ‘The line is a quarter wave length long, therefore: Z,, = Z? /R, = 25.2 168 Chapter Communication Systems b, The length of the shorted stub L, is such that reactance of type opposite to line is required, ‘On the Smith chart start at U, (Y'fuuf Fy = ©), move CCW to U where the imaginary component of G + jB is opposite that of the stub Jocation but equal in magnitude. Initial location o Final location 270° Difference 270° ‘The length of the stub: 270°/2 = 135° ¢, To determine the VSWR on the unmatched portion of the line L,, from point C on the Smith chart, we obtain N=2.6 This value can also be obtained as follows: Z, = 04+ j0.2 Ip| =Vo4" +02" = 0.486 1+[p] _ 140.446 _ 1.446 vswR= = = =26qec I-[p| 1-0,446 0.554 d %= 30040 Maiched load 2 = 30040 Eq = 800¥ (open creui) Vieng = 5002 = 250 Junction of L, & L; Exhibit 8.13¢ The power then will be supplicd to the load under the matched condition, hax = Vysaa(l+ | p |) = 250x 1 .446-= 360 volts Soutions 169 Exhibit 8.130 8.14 ae BG _(6x107 10° a ax PG 6x10° 2 = i eee ee ee surface area of sphere 47(50 10°)? 91% 10° watts/n® Prova = Alwin? @ SOKM] = 28.65 x 1.91 x 107 = 5.47 x 107 watt = 28.65 m’ effective antenna area Ane? a 4 Path Loss = -10 log G =Ilog $3 47") = 100g mx 50x 10° 6x! =—140log!.05 = 140.4 dB =-Wlog 20log 1.05 x 10? 70 (Chapter 8 Communication Systems 8.15 8.16 a The rms value of the resultant clectric field is given by the formula ttc anine$] where Sis the phase angle between /, and J. and is positive when f, leads fy nis the distance between A and & in wavelengths, and is usually frac- tional. Angle @ is measured from the normal to the line of the antennas ‘Thus, £,=¢,,,c05] “sing +™ ee N2 af For maximum é,, am[ Saino 7) me) « [fsnvet)o sing 205, 9=30° 1860. AL = 90" on{Esino"= 2) = cos = 0,707. 2 4 4 ‘The field strength of the array at 25 kM is «= 0.707250" [2| ™ 3 ‘The in-phase input is as follows: (A cos a9 cosas) = “Pfoor(w, 0, cose, +0, Wh ‘The 90° phase shift is as follows: (4 sin a9(¢ sineng) » A? feosten, —02,)!-+ costo, +4, ‘The sum of above two expressions is AG COs(OX — Wa). 72 (Chapter 8 Communication Systems b, Denoting the audio wave frequency with @s, we obtain vo(t)=A(L+ 0.Scosayt + 0.4 c08e),!)cos0a,F 5A 5 009(0, — 0) = Acosea,t + O54 cose, +09, t+ + 2A costa + aay + 24 - S cost, 0) Then the effective radiated power i Aa 2(osef (eee) rascal ° J] 2 = 10.125 +0,72 = 10,845 kW. ‘The frequency spectrum is as shown in Exhibit 8, Ye =o yo, % mre, yr O, ‘Exhibit 8.17 8.18 The number of different possible pictures is: 400 = gh 2e0* P= 10x 10...x 105 Probability of each element or picture is - Jt. gies ‘The channel capacity is defined as C= Lim log, MIT) where M(T is the total number of messages in T' seconds. =Slog,P bitlsee where Sis the signaling speed, 5 5 log, 10241 8 10 log, 10 = 6.0 x 10° bitlsec 8.19 a Soktions 173 Assumption: It is assumed that the signal-to-noise ratio is large or the “This is the single-stage, single-server queuing model with Poisson arriv- als and exponential service, Any basic operations research text gives the desired queue equations (e.g. Sasieni, Yaspan, and Friedman: Operations: Research-Methods aad Problems, John Wiley, pp. 126-138). mean arrival rate 4 = 10 wansmissions/hour mean service time = Vg = 4 minutes = 1/5 hour mean service rate pf = 15 recordings/hour. Average waiting time of an arrival 10 =10 hour =8 minutes Eon =— = ™ Hit=Ay 1SIS=10) 75 the single-stage, 1wo-server queuing situation. The various expec- tations for this model may also be obtained from a basic operations research (or queuing} text. Before procecding, a quick check can be made. We know a second recorder will substantially reduce (but not eliminate) waiting time, If the elimination of waiting time would not justify the second recorder then. We need not bother to make the exact computation, Instead, we could simply conclude the second recorder is not economically warranted. Hourly saving (assuming elimination of waiting time) = mean arrival rate x mean waiting time reduction * line charge = ALE Ge.) = B(w;)] 0.04 = (10)(8 = 040.04) = $3.20 Hourly cost = $2.50 ‘Thus we have been unable to show that the second recorder is uneco- nomical at vero waiting time. We must, therefore, proceed to compute the expectation of average waiting time. satay|+ald) For two service facilities k=2 A= pois 174 = Chanter Communication Systems. Average waiting time of an arrival savings exceeds the hourly cost. RECOMMENDED REFERENCES Sasieni, Yaspan, and Friedman, Operations Rescarch-Methods and Problems, John Wiley, 1964, pp. 126-138, Schwartz. Information Transmission, Modulation and Noise, McGraw-Hill, 1990, pp. 106, 107. Stevenson, William D. Jr, Elements of Power System Analysis, McGraw-Hill, 1982, pp. 100-106, Also see the 1994 revised edition referenced al the end of Chapter 3 CHAPTER |— Biomedical Instrumentation and Safety OUTLINE PROBLEMS 175 SOLUTIONS 177 PROBLEMS 9.1 The output of a pair of electrocardiograph skin electrodes is to be amplified with the differential instramentation amplifier shown in Exhibit 9.1, The imum peak differential signal amplitude expected from the electrodes is 0.5mV, The maximum peak differential signal araplitude expected from the electrodes is 4.0 m¥. The minimum peak signal amplitude desired from the differential amplifier output is 0.5V. The gain of the output amplifier stage from V, and V, to ¥, is desired to be-40 times the gain of the output amplifier state from V~ and V* to V, and Vp 75 76 Chapter 9 Biomedical instrumentation and Safety 92 93 The differential gain of the entire amplifier (Gp) is the differential gain of the output stage (Gy...) times the differential gain of the input stage (Gp) Of Gy = Grae’ Grin a. IPR, is 100KQ, Ry is 1OKQ, and R, is 10k, what are the values of Ry, Ry. Rs, and Ry? Assume ideal op-amps. b. If the op-amps are capable of producing an output voltage within 1 V of the power supply voltage, what should the power supply voltage be so thal the maximum peak signal amplitude from the differential ampli- fier output is not clipped? For the differential instrumentation amplifier shown in Exhibit 9.1 a, What is the best CMRR that can be achieved using +10% tolerance: resistors, if all seven resistors were desired to be identical? Assume ideal op-amps. b. What is the minimum resistor tolerance necessary to achieve a CMRR of at least 100 dB? Assume ideal op-amps. Exhibit 9.2 shows a microshock circuit. Guatnge Pearse Fos 120 VAG A, Frans Exhibit 9.2 a. fan isolation amplifier is inserted between the pressure transducer and the power supply, what is the minimum value of the series isolation resistance necessary to reduce the leakage current to one tenth of the maximum safe current limit? b. The engineer installing the isolation amplifier is concerned about saline solution from the catheter being spilled on the isolation amplifier cir- cuitry and thus bridging the isolation protection with a low impedance leakage path. The isolation amplifier is 10 mm wide, and 20 mm long from the input terminals to the outpat terminals. ‘A.column of saline solution has cross-sectional resistance per length of about 200 & mm'/mm for cach mm of column length per each mm? of column surface area. Thus, the total resistance of a saline column is (200.2 mm)- (saline column length in mmy(saline column cross-sectional area in mm*). How deep would a layer of saline solution spanning the isolation amplifier have to be for the leakage current to reach the maximum safe microshock current limit? c. The engincer installing the isolation amplifier decides to encapsulate the amplifier with a plastic that has a primarily dielectric insulation characteristic. The leakage current through the effective encapsulation capacitor betwcen the input and output of the isolation amplifier is to be no greater that the leakage current through the isolation amplifier. ‘What is the maximum value of the encapsulation capacitor? Engineering Economics PROBLEMS Al Ad A3 Ad AS A loan was made 24% years ago at 8% simple annual interest. The principal amount of the loan has just been repaid along with $600 of interest. The principal amount of the loan was closest to a $300 c. $4000 b. $3000 d. $5000 A $1000 Joan was made at 10% simple annual interest. It will take how many years for the amount of the loan and interest to equal $1700? a. 6 years: c. 8 years b. 7 years d. 9 years A retirement fund ears 8% interest, compounded quarterly. If $400 is deposited every three months for 25 years, the amount in the fund at the end of 25 years is nearest to a. $50,000 c. $100,000 b. $75,000 d. $125,000 For some interest rate /, and some number of interest periods, the uniform ‘series capital recovery factor is 0.2091 and the sinking fund factor is 0.1941. The interest rate ¢ must be closest to al c. 3% b. 28% d. 4% ‘The repair costs for some handheld equipment is estimated to be $120 the first year, increasing by $30 per yeur in subsequent years. The amount a person will need to deposit into a bank account paying 4% interest to provide for the repair costs for the next five years is nearest to a. $500 c. $700 b. $600 d. $800 170 APPENDIX ;-— AS Al6 ALT ALB ALD AW A2l A.22 Problems 181 If 10% nominal annual interest is compounded daily, the effective annual interest rate is nearest 10 a. 10.00% <, 10,50% b. 10.38% a. 10.75% If 10% nominal annual interest is compounded continuously, the effective annual interest rate is nearest to a. 10.00% © 10.50% b. 10.38% a. 10.75% Ir the quarterly effective interest rate is 5'/:% with continuous compound- ing, the nominal interest rate is nearest to a 55% <. 16.5% b 110% 4. 21.4% A continuously compounded loan has what effective interest rate if the ntominal iiiterest rate is 25%? Was 03s ae’ a Oa 02s b ¢ d, In(.25) A contiquously compounded foan has what nominal interest rare if the effective interest rate is 25%? . c. In(E.25) b logy 1.25) An individual wishes to deposit a certain quantity of money now so that he will have $500 at the end of five years, With interest at 4% per year, compound semiannually, the amount of the depos Nearest to a $340 c S410 b, $400 d, $608 A steam boiler is purchased on the basis of guaranteed performance. A test indicates that the operating cost will be $300 more per year than the manufacturer guaranteed. If the expected life of the boiler is 20 years, and money is worth 8%, the amount the purchaser should deduct from the purchase price to compensate For the extra operating cost i5 nearest 10 a $2950 c $4100 b $3320 4, $5520 A consulting engineer bought a fax machine with one year's free main- tenance, fn the second year the maintenance is estimated at $20, In subsequent years the maintenance cost will increase $20 per year (that is, third year maintenance will be $40, fourth year maintenance will be $60, and so forth). The amount that must be set aside now at 6% interest to pay the maintenance costs on the fax machine for the first six years of ownership is nearest to a, $101 <, $229 b, S164 d, $284 182 Append A Ergincening Eoonomics A23 Au AS A26 AB A299 An investor is considering buying a 20-year corporate bond. The bond has a face value of § 1000 and pays 6% interest per year in two semiannual payments, Thus the purchaser of the bond will receive $30 every six ‘months, and in addition he will reccive $1000 at the end of 20 years, along ‘with the last $30 interest payment, If the investor believes he should receive 8% annual interest, compounded semiannually, the amount he is willing to pay for the bond value of closesi to a. $500 c. $800 b. $600 dd. $900 Annual maintenance costs for a particular section of highway pavement are 32000. The placement of a new surface would reduce the annual mainte- nance cost to $500 per year for the first five years and to $1000 per year for the next five years. The annual maintenance after 10 years would again be $2000. If maintenance costs are the only saving, the maximum investment that can be justified for the new surface, with interest at 4%, is closest to a $5,500 c. $10,000 b, $7,170 4. $10,340 A project has an initial cost of $10,000, uniform annual benefits of $2400, and a salvage value of $3000 at the end of its 10-year useful life. At 12% interest the net present warth of the project is closest to- a $2,500 c. $4,500 b. $3,500 d, $5,500 A person barrows $5000 at an interest rate af 18%, compounded monthly. Monthly payments of $167.10 are agreed upon. The length of the loan is closest to a. 12 months c. 24 months 20 months d. 40 months A machine costing $2000 to buy and $300 per year to operate will save labor expenses of $650 pet year for eight years. The machine will be purchased if its salvage value at the end of eight years is sufficiently large to make the investment economically attractive. If an interest rate of 10% is used, the minimum salvage value must be closest to a $100 <. $300 b $200 d. $400 ‘The amount of shoney deposited 50 years ago al 89% interest that would now provide a perpetual payment of $10,000 per year is nearest to- a. $3,000 ¢, $50,000 b. $8,000 4. $70,000 An industrial firm must pay a local jurisdiction the cost to expand its sewage treatment plant. In addition, the firm must pay $12,000 annually toward the plant operating costs. The industrial firm will pay sufficient money into a fund that earns 5% per year, to pay its share of the plant operating costs forever. ‘The amount to be paid to the fund is nearest to a. $15,000 <, $160,000 b. $60,000 d. $240,000 A30 AQ A32 AB AM ASS A386 Problems 183 At an interest rate of 25% per month, money will double in value in how many months? a, 20 months c. 50 months b. 35 months d. 65 months A woman deposited $10,000 into an account at her credit union. The: money was left on deposit for 80 months, During the first 50 months the woman carned 12% interest, compounded monthly, The credit union then changed its interest policy so that the woman earned 8% interest cam- pounded quarterly during the next 30 months. The amount of money in the account at the end of 80 months is nearest to a, $10,000 ec. $20,000 b. $15,000 d. $25,000 An engineer deposited $200 quarterly in her savings account for three ‘years at 6% interest, compounded quarterly. Then for five years she made no deposits or withdrawals. The amount in the account after eight years is closest to a, $1200 ec, $2400 b. $1800 d. $3600 A sum of money, Q, will be received six years from now, At 6% annual interest the present worth now of @ is $60, At this same interest rate the value of O 10 years from now is closest to a. $60 c, $90 b. 877 d. $107 If $200 is deposited in a savings account at the beginning of each of 15 years and the aecount ears interest at 6%, compounded annually, the value of the account at the end of 15 years will be most nearly a. $4500 ec. $4900 b. $4700 a. ssi00 The maintenance expense on a piece of machinery is estimated as follows: Year 1 2 3 4 Maintenance «S150, S300 SAS S600) If interest is 8%, the equivalent uniform annual maintenance cost is closest to a. $250 c. $350 b. $300 d. $400 ‘A payment of $12,000 six years from now is equivalent, at 10% interest, to an annual payment for eight years starting at the end of this year, The annual payment is closest to a. $1000 e. $1400 b. $1200 d. $1600 184 — Appancee A Engneenng Economics ABT A39 AaO AaL AA? A manufacturer purchased $15,000 worth of equipment with a useful life cof six years and a $2000 salvage value at the end of the six years. Assuming a 12% interest rate, the equivalent uniform annual cost is nearest to a, $1500 cc. $3500 b. $2500 a. $4500 ‘Consider a machine as follows: Initial cost: $80,000 End-of-useful-life salvage valuc: $20,000 Annual operating cost: $18,000 ‘Useful life: 20 years Based on 10% interest, the equivalent uniform annual cost for the machine is closest to a. $21,000 c. $25,000 bh, $23,000 a. $27,000 Consider a machine as follows: Initial cast: $80,000 Annual operating ¢ost: $18,000 Useful life: 20 years ‘What must be the salvage value of the machine at the end of 20 years for the machine to have an equivalent uniform annual cost of $27,000? Assume a 10% interest rate. The: salvage value is closest to a. $10,000 c. $40,000 b. $20,000 d. $50,000 ‘Twenty-five thousand dollars is deposited in a savings account that pays 5% interest, compounded semiannually. Equal annual withdrawals are to be made from the account beginning one year from now and continuing forever. The maximum amount of the equal annual withdrawals is closest to a $625 c. $1250 b. $1000 4. $1265 An investor is considering investing $10,000 in a piece of land. The property taxes are $100 per year. The lowest selling price the investor must receive if she wishes to earn a 10% interest rate after keeping the land for 10 years is a. $21,000 c. $27,000 b. $23,000 4. $31,000 ‘The rate of return of a $10,000 investment that will yield $1000 per year for 20 years is closest to a 1% c. 8% b 4% 4d. 12% AAR As Ad5 AAG AAT Problems 185 ‘An engincer invested $10,000 in a company, In retum he received $600 per year for six years and his $10,000 investment back at the end of the six years. His rate of ret on the investment was closest to a. 6% ec. 12% b. 10% d. 15% An engineer made 10 annual end-of-year purchases of $1000 of common stock. At the end of the tenth year, just after the last purchase, the engineer sold all the stock for $12,000, The rate of retum received on the investment is closest to a 2% ce. 8% b. 4% d. 10% A company is considering buying a new piece of machinery. Initial cost: $80,000 End-of-useful-tife salvage value: $20,000 ‘Annual operating cost: $18,000 Useful fe: 20 years ‘The machine will produce an annual savings in material of $25,700. What is the before-tax rate of return if the machine is installed? The rate of retum is closest to a 6% c. 10% b. 8% d. 15% Consider the following situation: Invest $100 now and receive two pay ments of $102.15—one at the end of Year 3, and one at the end of Year 6, ‘The rate of return is nearest to a 8% c. 18% b 12% d, 22% ‘Two mutually exclusive alternatives are being considered: A B 0 S250) = $8000 IL +746, +1664 2 +746 +1664 3 +746 +1664 4 +746 +1664 s +746 +1664 The tate of return on the difference between the alternatives is closest to a 6% ce. 10% b 8% ad 12% AS ASA ASO AST Given two machines: A B Initial cot $55,000 $75,000 Total annual costs $16,200 812.450 With interest at 10% per year, at what service life do these two machines have the same equivalent uniform annual cost? The service life is closest to a. 5 years eT years b. 6 years d. 8 years A machine part that is operating in a corrosive atmosphere is made of low-carbon steel. It costs $350 installed, and lasts six years. If the part is treated for corrosion resistance it will cost $700 installed. How long must the treated part last to be as economic as the untreated part, if money is worth 6%? a & years c. 15 years b. 11 years d, 17 years A firm has determined the two best paints for its machinery are Tuff-Coat at $45 per gallon and Quick at $22 per gallon. The Quick paint is expected to prevent rust for five years. Both paints take $40 of labor per gallon to apply, and both cover the same area, If a 12% interest rate is used, how long must the Tuff-Coat paint prevent rust to justify its use? a 5 years ce. 7 years b. 6 years d. 8 years ‘Two altematives are being considered: A B Cost 31000 $2000. Useful life in years to to End-of-uceful-tife salvage value 100 400 ‘The net annual benefit of A is SISO. If interest is 8%, what must be the net annual benefit of B for the two alternatives to be equally desirable? ‘The net annual benefit of B must be closest to a. $150 ce. $225 b. $200 d. $275 Which one of the following is NOT a method of depreciating plant equip- ment for accounting and engineering economics purposes? a. double-entry method b. modified accelerated cost recovery system c. sum-of-years-digits method d. straight-line method

You might also like